SlideShare a Scribd company logo
1 of 41
Download to read offline
Solu¸˜es dos Exerc´
       co            ıcios da Se¸˜o 4.1
                                ca

1) A primeira pergunta pode ser respondida de 5 modos; a segunda, de 5
   modos, etc.
   A resposta ´ 5 × 5 × · · · × 5 = 510 = 9.765.625.
              e

2) Para formar um subconjunto, deve-se decidir, para cada elemento do
   conjunto, se ele pertencer´ ou n˜o ao subconjunto. H´ 2 modos de
                              a     a                     a
   decidir o que fazer com o primeiro elemento do conjunto, 2 modos com
   o segundo, etc.
   A resposta ´ 2 × 2 × · · · × 2 = 2n .
              e
   Outra solu¸˜o:
             ca
   Quando se acrescenta um elemento a um conjunto, os subconjuntos do
   novo conjunto s˜o os subconjuntos do conjunto original e estes unidos
                   a
   ao novo elemento. Ou seja, o n´ mero de subconjuntos dobra. Ent˜o,
                                  u                                  a
   se An ´ o n´ mero de subconjuntos de um conjunto com n elementos,
           e   u
   (An ) ´ uma progress˜o geom´trica de raz˜o 2. Logo, An = A0 .2n = 2n
         e             a        e          a
   pois o conjunto vazio possui um unico subconjunto.
                                   ´

3) A primeira pessoa tem 5 escolhas; a segunda, 4; a terceira, 3. A resposta
   ´ 5 × 4 × 3 = 60.
   e

4) Os bancos em que os homens se sentam podem ser escolhidos de 5×4×
   3×2×1 = 120 modos, o mesmo ocorrendo com os bancos das mulheres.
   Em cada banco, os casais podem se sentar de 2 modos diferentes. A
   resposta ´ 1202 × 25 = 460.800.
            e

5) As 64 casas do tabuleiro dividem-se, naturalmente, em trˆs grupos:
                                                           e
   i) as 4 casas dos v´rtices;
                      e
   ii) as 24 casas da borda do tabuleiro, mas que n˜o s˜o v´rtices;
                                                   a a e
   iii) as restantes 36 casas, que s˜o interiores ao tabuleiro.
                                    a
   Vamos separar a nossa contagem conforme o tipo de casa ocupada pelo
   rei negro:
   i) h´ 4 possibilidades para o rei negro e 60 para o rei branco;
       a
   ii) h´ 24 possibilidades para o rei negro e 58 para o rei branco;
        a
iii) h´ 36 possibilidades para o rei negro e 55 para o rei branco.
         a
   A resposta ´ 4 × 60 + 24 × 58 + 36 × 55 = 3612.
              e
   Se os reis s˜o iguais, a resposta passa a ser a metade da resposta an-
                a
   terior, pois, trocando a posi¸˜o dos reis, agora obt´m-se a mesma con-
                                ca                     e
   figura¸˜o.
         ca

6) Haver´ uma torre em cada linha e em cada coluna. A posi¸˜o da torre
         a                                                 ca
   da primeira linha pode ser escolhida de 8 modos; a da segunda linha,
   de 7, etc.
   A resposta ´ 8 × 7 × 6 × 5 × 4 × 3 × 2 × 1 = 40.320.
              e
   Se as torres fossem diferentes, para cada uma das escolhas de posi¸˜o,
                                                                     ca
   ter´
      ıamos que escolher uma das torres. A resposta seria, portanto,
   8 × 8 × 7 × 7 × 6 × 6 × 5 × 5 × 4 × ×3 × ×3 × 2 × 2 × 1 × 1 = (8!)2 =
   1.625.702.400.

7) Se a primeira carta ´ o rei de copas, a segunda pode ser escolhida de 48
                       e
   modos (pode ser qualquer carta, exceto os 4 reis). Se a primeira carta ´e
   de copas mas n˜o ´ o rei, ela pode ser escolhida de 12 modos. Neste caso,
                  a e
   a segunda carta pode ser escolhida de 47 modos (n˜o pode ser a primeira
                                                        a
   escolhida, nem nenhum dos 4 reis). A resposta ´ 48 + 12 × 47 = 612.
                                                      e

8) a) Para construir uma fun¸˜o, devemos, para cada elemento de A, es-
                            ca
   colher sua imagem em B. H´ 7 modos de escolher a imagem do primeiro
                            a
   elemento de A, 7 modos de escolher a imagem do segundo elemento,
   etc.
   A resposta ´ 7 × 7 × 7 × 7 = 2.401.
              e
   b) Para a fun¸˜o ser injetora, elementos diferentes devem ter imagens
                 ca
   diferentes. H´ 7 modos de escolher a imagem do primeiro elemento de
                a
   A, 6 modos de escolher a imagem do segundo elemento, etc.
   A resposta ´ 7 × 6 × 5 × 4 = 840.
              e

9) a) Como 720 = 24 × 32 × 51 , 720 possui 5 × 3 × 2 = 30 divisores. Aos
   pares, estes divisores formam produtos iguais a 720. Logo, h´ 15 modos
                                                               a
   de escrever 720 como um produto de divisores.
   b) Como 144 = 24 × 32 , 144 possui 3 × 3 = 9 divisores. Com eles,
   podem ser formados 4 pares de divisores cujo produto ´ 144 e, al´m
                                                        e          e

                                    2
disso, pode ser formado o produto 12 × 12. Assim, h´ 5 modos de
                                                        a
     escrever 144 como um produto de divisores.

10) Um arm´rio ficar´ aberto se ele for mexido um n´mero ´
            a             a                               u      ımpar de vezes.
    Por outro lado, o arm´rio de ordem k ´ mexido pelas pessoas cujos
                                   a              e
    n´ meros s˜o divisores de k. Logo, estar˜o abertos os arm´rios cujos
     u        a                                    a                 a
    n´ meros possuem um n´ mero ´
     u                               u   ımpar de divisores. Isto ocorre com os
    n´ meros cujos expoentes s˜o todos pares na decomposi¸˜o em fatores
     u                                 a                          ca
    primos, ou seja, s˜o quadrados perfeitos. Assim, permanecer˜o aber-
                            a                                          a
    tos os arm´rios cujos n´ meros s˜o quadrados perfeitos, ou seja, os de
              a                      u    a
    n´ meros 12 , 22 , . . . , 302 .
     u

11) Separemos o caso em que o primeiro e o terceiro quadrantes tˆm cores
                                                                e
    iguais do caso em que eles tˆm cores diferentes.
                                e
     No caso de cores iguais, h´ 5 modos de escolher a cor unica para o
                                a                             ´
     primeiro e o terceiro quadrantes, 4 modos de escolher a cor para o
     segundo quadrante e 4 modos de escolher a cor para o quarto quadrante.
     H´, portanto, 5 × 4 × 4 = 80 modos de colorir o mapa usando cores
       a
     iguais no primeiro e no terceiro quadrantes.
     No caso de cores diferentes, h´ 5 modos de escolher a cor para o primeiro
                                   a
     quadrante, 4 modos de escolher a cor para o terceiro quadrante, 3 modos
     de escolher a cor para o segundo quadrante e 3 modos de escolher a cor
     para o quarto quadrante. H´ 5 × 4 × 3 × 3 = 180 modos de colorir o
                                   a
     mapa usando cores iguais no primeiro e no terceiro quadrantes.
     No total, temos, portanto, 80 + 180 = 260 modos de colorir a figura.

12) a) H´ 265 = 11.881.376 palavras de 5 letras. Delas, devemos subtrair
         a
    as palavras que come¸am por A, 1 × 264 = 456.976, e aquelas nas quais
                         c
    a letra A n˜o figura, 255 = 9.765.625.
               a
     A resposta ´ 11.881.376 − 456.976 − 9.765.625 = 1.658.775.
                e
     b) O n´ mero total de palavras de 5 letras distintas ´ 26 × 25 × 24 ×
            u                                             e
     23 × 22 = 7.893.600. Delas devemos subtrair as palavras que come¸am
                                                                       c
     por A, 1 × 25 × 24 × 23 × 22 = 303.600 e aquelas nas quais a letra A
     n˜o figura, 25 × 24 × 23 × 22 × 21 = 6.375.600.
      a
     A resposta ´ 7.893.600 − 303.600 − 6.375.600 = 1.214.400.
                e

     Outra solu¸˜o:
               ca

                                      3
H´ 4 posi¸˜es para colocar a letra A; depois disso, as quatro casas
      a       co
    vazias podem ser preenchidas de 25, 24, 23 e 22 modos.
    A resposta ´ 4 × 25 × 24 × 23 × 22 = 1.214.400.
               e

13) Cada letra pode ser escolhida de 26 modos, enquanto cada algarismo
    pode ser escolhido de 10 modos. Logo, o n´ mero total de placas ´
                                               u                     e
    263 × 104 = 175.760.000.

14) O n´ mero de modos de acomodar os passageiros que pretendem sentar
        u
    de frente ´ 5 × 4 × 3 × 2 = 120; o n´ mero de modos de acomodar os
              e                         u
    passageiros que pretendem sentar de costas ´ 5 × 4 × 3 = 60; o n´mero
                                               e                    u
    de modos de acomodar os demais passageiros ´ 3 × 2 × 1 = 6.
                                                  e
    A resposta ´ 120 × 60 × 6 = 43.200.
               e

15) Vamos primeiramente determinar quantos zeros s˜o escritos na casa
                                                  a
    das unidades, depois na das dezenas, etc.
    H´ 222 n´ meros que tˆm 0 como algarismo das unidades, pois antes do
      a     u            e
    zero podem ser colocados os inteiros de 1 (inclusive) a 222 (inclusive).
    H´ 22 × 10 = 220 n´meros que tˆm 0 como algarismo das dezenas,
      a                   u              e
    pois antes do zero podem ser colocados os inteiros de 1 (inclusive) a 22
    (inclusive) e depois do zero, os inteiros de 0 (inclusive) a 9 (inclusive).
    H´ 2 × 100 = 200 n´meros que tˆm 0 como algarismo das centenas,
      a                   u              e
    pois antes do zero podem ser colocados os inteiros de 1 (inclusive) a 2
    (inclusive) e depois do zero, os inteiros de 0 (inclusive) a 99 (inclusive).
    A resposta ´ 222 + 220 + 200 = 642.
               e
    ´
16) E mais simples contar, primeiramente, os n´ meros onde o algarismo
                                                u
    5 n˜o aparece. O primeiro d´
       a                        ıgito pode ser escolhido de 8 modos (n˜o
                                                                      a
    pode ser igual a 0 nem igual a 5) e cada um dos demais trˆs d´
                                                               e ıgitos
    pode ser selecionado de 9 modos (deve ser diferente de 5). Logo, h´ a
    8×93 = 5.832 n´ meros de 4 algarismos em que n˜o aparece o algarismo
                    u                               a
    5.
    A quantidade de n´ meros de 4 d´
                       u            ıgitos, com ou sem o d´ıgito 5, ´ 9×103 =
                                                                    e
    9.000 (pois h´ 9 modos de selecionar o primeiro d´
                  a                                      ıgito, que deve ser
    diferente de 0, e 10 modos de selecionar cada um dos demais 4 d´   ıgitos).
    Logo, h´ 9.000 − 5.832 = 3.168 n´ meros de 4 algarismos em que o 5
            a                           u
    n˜o aparece.
     a

                                      4
17) Devemos decidir quantos exemplares de cada revista devem ser postos
    na cole¸˜o. H´ 6 possibilidades para a ”Veja” (0, 1, 2, 3, 4, ou 5
           ca      a
                                                        ´
    exemplares), 7 para a ”Manchete” e 5 para a ”Isto E”. O n´ mero de
                                                                u
    cole¸˜es ´ 6 × 7 × 5 = 210, e o n´mero de cole¸˜es n˜o-vazias ´ 209.
        co e                         u            co    a         e

18) Em cada dia, duas das mat´rias s˜o ensinadas e uma folga. H´ 3
                                  e       a                           a
    possibilidades para escolher a mat´ria que folga na segunda, 2 para
                                         e
    escolher a que folga na quarta e 1 para escolher a que folga na sexta.
    Portanto, h´ 6 modos para escolher as mat´rias de cada dia. Para
                a                                 e
    escolher os hor´rios, h´ 2 possibilidades em cada dia. Logo, o n´mero
                   a       a                                        u
    total de hor´rios ´ 6 × 8 = 48.
                a     e

19) Foi feita uma distin¸˜o artificial ao se considerar cada casal ordenado
                         ca
    de dois modos diferentes: come¸ando pela mulher ou pelo homem. Por
                                   c
    esta raz˜o, o resultado encontrado foi igual ao dobro do correto.
             a

20) H´ trˆs tipos de cart˜es: os que virados de cabe¸a para baixo n˜o
      a e                  o                            c                a
    representam n´ meros, como, por exemplo, 41.809; os que virados de
                   u
    cabe¸a para baixo representam o mesmo n´mero, como, por exem-
         c                                       u
    plo, 86.198; os que virados de cabe¸a para baixo representam n´meros
                                       c                            u
    diferentes, como, por exemplo, 66.810. Os cart˜es do ultimo tipo s˜o
                                                    o       ´            a
    os que permitem economia porque um mesmo cart˜o serve para rep-
                                                        a
    resentar dois n´ meros. H´ 5 × 5 × 5 × 5 × 5 = 3.125 cart˜es que
                    u           a                                   o
    virados de cabe¸a para baixo representam n´meros, iguais ou difer-
                     c                            u
    entes, pois tais cart˜es devem ter como algarismos apenas 0, 1, 8, 6
                         o
    ou 9. Destes, 5 × 5 × 3 = 75 s˜o do segundo tipo, pois um tal cart˜o
                                    a                                    a
    deve ter as casas das extremidades preenchidas por 00, 11, 88, 69 ou
    96, a segunda e a quarta casas preenchidas por 00, 11, 88, 69 ou 96, e a
    casa central preenchida por 0, 1 ou 8. Portanto, os cart˜es do terceiro
                                                             o
    tipo s˜o em n´ mero de 3.125 − 75 = 3.050. Podem ser economiza-
           a       u
    dos 3.050/2 = 1.525 cart˜es. O n´ mero m´
                              o        u       ınimo de cart˜es de que se
                                                              o
    necessita ´ 100.000 − 1.525 = 98.475.
              e

21) A decomposi¸˜o de 360 em fatores primos ´ 720 = 23 .32 .5. Os divisores
                   ca                               e
    inteiros e positivos de 720 s˜o os n´ meros da forma 2α .3β .5γ , com α ∈
                                  a        u
    {0, 1, 2, 3}, β ∈ {0, 1, 2} e γ ∈ {0, 1}. A soma dos divisores ´ S =    e
       α β γ
    Σ2 .3 .5 , com α ∈ {0, 1, 2, 3}, β ∈ {0, 1, 2} e γ ∈ {0, 1}. Para calcular
    essa soma, dividimos as parcelas em dois grupos, conforme seja γ = 0 ou
    γ = 1. S = Σ (2α .3β .50 ) + Σ (2α .3β .51 ) = 6 Σ (2α .3β ) porque a segunda

                                       5
soma ´ igual ao qu´
          e                ıntuplo da primeira. Agora, dividimos as parcelas
   em grupos, conforme seja β = 0, β = 1 ou β = 2. S = 6 [Σ (2α .30 ) +
   Σ (2α .31 ) + Σ (2α .32 )] = 6[Σ 2α + 3Σ 2α + 9Σ 2α ] = 6[13Σ 2α ] = 78Σ2α =
   78[20 + 21 + 22 + 23 ] = 78 × 15 = 1.170.



   Solu¸˜es dos Exerc´
       co            ıcios da Se¸˜o 4.2
                                ca

1) a) O n´ mero total de anagramas ´ 8! = 40.320.
         u                         e
   b) H´ 4 modos de escolher a vogal que ser´ a primeira letra do anagrama
        a                                   a
   e 3 modos de selecionar a vogal que ser´ a ultima letra do anagrama.
                                            a ´
   Depois disso, h´ 6! modos de arrumar as demais letras entre a primeira
                  a
   e a ultima.
       ´
   A resposta ´ 4 × 3 × 6! = 4 × 3 × 720 = 8.640.
              e
   c) As vogais e consoantes podem aparecer na ordem CV CV CV CV
   ou na ordem VC VC VC VC. No primeiro caso, devemos colocar as 4
   vogais nos 4 lugares de ordem par (4! modos) e as 4 consoantes nos 4
   lugares de ordem ´ımpar (4! modos).
   H´ 4!×4! = 24 ×24 = 576 anagramas do primeiro tipo. Analogamente,
    a
   h´ 576 anagramas do segundo tipo.
    a
   A resposta ´ 576 + 576 = 1.152.
              e
   d) Tudo se passa como se CAP fosse uma unica letra. Devemos, por-
                                            ´
   tanto, arrumar em fila 6 objetos: CAP,I,T,U,L,O.
   A resposta ´ 6! = 720.
              e
   e) Primeiramente, devemos escolher a ordem em que as letras C, A, P
   aparecer˜o. H´ 3! modos. Depois, devemos arrumar em fila 6 objetos:
           a     a
   o bloco das letras C, A, P e as 5 letras I, T, U, L, O. H´ 6! modos.
                                                            a
   A resposta ´ 3! × 6! = 6 × 720 = 4320.
              e
   f) Basta arrumar em fila, depois do PA, as restantes 6 letras.
   A resposta ´ 6! = 720.
              e
   g) H´ 7! anagramas com a letra P em primeiro lugar e h´ 7! anagramas
       a                                                 a
   com a letra A em segundo lugar. H´ tamb´m 6! anagramas com P
                                       a      e
   em primeiro lugar e A em segundo lugar. Ao somarmos 7! com 7!,

                                     6
encontramos o n´ mero de anagramas com P em primeiro lugar ou A
                   u
   em segundo lugar, mas contamos duas vezes os anagramas que tˆm P
                                                                 e
   em primeiro lugar e A em segundo lugar. A resposta ´ 7! + 7! − 6! =
                                                      e
   5040 + 5040 − 720 = 9.360.
   h) H´ 7! anagramas com a letra P em primeiro lugar, 7! anagramas
        a
   com a letra A em segundo e 7! anagramas com a letra C em terceiro.
   H´ tamb´m 6! anagramas com P em primeiro lugar e A em segundo
     a       e
   lugar, 6! anagramas com P em primeiro e C em terceiro e 6! anagramas
   com A em segundo e C em terceiro. Finalmente, h´ 5! anagramas com
                                                   a
   P em primeiro lugar, A em segundo e C em terceiro.
   Ao somarmos 7! com 7! com 7!, encontramos o n´mero de anagramas
                                                u
   que tˆm P em primeiro lugar ou A em segundo ou C em terceiro, mas
        e
   contamos alguns anagramas v´rias vezes.
                               a
   Contamos duas vezes os anagramas que tˆm P em primeiro lugar e A em
                                           e
   segundo; o mesmo se deu com os que tˆm P em primeiro e C em terceiro
                                       e
   e com os que tˆm A em segundo e C em terceiro. Descontando essas
                 e
   contagens indevidas, chegamos a 7! + 7! + 7! − 6! − 6! − 6! = 3.7! − 3.6!.
   Entretanto, anagramas com P em primeiro lugar e A em segundo e C
   em terceiro foram, inicialmente, contados trˆs vezes e, posteriormente,
                                                e
   descontados trˆs vezes, o que significa que n˜o est˜o sendo contados.
                   e                              a    a
   Incluindo-os na contagem, obtemos a resposta correta, que ´ 3.(7!) −
                                                                 e
   3.(6!) + 5! = 3.(5.040) − 3.(720) + 120 = 13.080.
   i) Como h´ 6 ordens poss´
              a            ıveis para as letras C, A e P, os anagramas
   pedidos s˜o exatamente 1/6 do total, ou seja, 8!/6 = 6.720.
            a
   Outra solu¸˜o:
             ca
   Basta escolher as 3 posi¸˜es a serem ocupadas pelas Letras P, A, C, o
                           co
                           3
   que pode ser feito de C8 = 56 modos e distribuir as 5 letras restantes
   nas demais posi¸˜es, o que pode ser feito de 5! = 120 modos. O total
                   co
   de anagramas ´ 56 × 120 = 6.720.
                 e

2) O valor de f (a1 ) pode ser escolhido de n modos; o valor de f (a2 ), de
   n − 1 modos; . . . ; o de f (an ), de 1 modo.
   A resposta ´ n(n − 1) . . . 1 = n!
              e

3) O n´ mero total de modos de sentar 8 pessoas em 8 cadeiras ´ o n´ mero
      u                                                       e    u
   de modos de arrumar 8 pessoas em fila, 8!. O n´mero de modos de
                                                    u

                                        7
arrumar 8 pessoas em fila de modo que duas dessas pessoas, Vera e
   Paulo, fiquem juntas ´ 2.7!, pois, para formar uma tal fila, devemos
                         e
   inicialmente decidir em que ordem se colocar˜o Vera e Paulo e, em
                                                a
   seguida, formar uma fila de 7 objetos: o bloco formado por Vera e
   Paulo; as demais 6 pessoas.
   A resposta ´ 8! - 2.7! = 40.320 - 10.080 = 30.240.
              e

4) Como visto no problema anterior, o n´mero de filas nas quais duas
                                         u
   pessoas (neste caso Helena e Pedro) ficam juntas ´ 2.7! = 10.080. O
                                                     e
   n´ mero de filas onde Helena e Pedro e tamb´m Vera e Paulo ficam
    u                                           e
   juntos ´ obtido de modo an´logo: agora s˜o dois blocos de duas pes-
          e                    a            a
   soas, cada um podendo ser arrumado de dois modos distintos e mais
   4 pessoas. Portanto, o n´mero de tais filas ´ 2.2.6! = 2.880. Logo, o
                            u                 e
   n´ mero de filas em que Helena e Pedro ficam juntos, mas Vera e Paulo
    u
   n˜o, ´ 10.080 − 2.880 = 7.200.
    a e

5) O elemento da permuta¸˜o que ocupa o 10o lugar deve ser maior que 7.
                            ca
   Pode ser escolhido de 3 modos. O elemento da 9a posi¸˜o deve ser maior
                                                          ca
   que 6; haveria 4 possibilidades, mas uma delas j´ foi usada na escolha
                                                      a
   do elemento que ocupa a 10a posi¸˜o. Pode ser escolhido de 3 modos.
                                        ca
   Prosseguindo com esse racioc´  ınio, vemos que a cada nova casa abranda-
   se a restri¸˜o, criando uma possibilidade a mais, mas ao mesmo tempo
              ca
   diminui-se uma possibilidade, pois uma delas foi usada na etapa. Ou
   seja, h´ 3 possibilidades para cada casa at´ a 3a casa. O elemento da
          a                                     e
   3a posi¸˜o deve ser maior que 3 − 3 = 0; haveria 10 possibilidades,
           ca
   mas 7 delas j´ foram usadas nas etapas anteriores. Pode ser escolhido
                 a
   de 10 − 7 = 3 modos. O elemento da 2a posi¸˜o deve ser maior que
                                                     ca
   2 − 3 = −1; haveria 10 possibilidades, mas 8 delas j´ foram usadas nas
                                                         a
   etapas anteriores. Pode ser escolhido de 10−8 = 2 modos. Finalmente,
   o elemento de posi¸˜o 1 deve ser maior que 1 − 3 = −2; haveria 10
                        ca
   possibilidades, mas 9 delas j´ foram usadas nas etapas anteriores. Pode
                                a
   ser escolhido de 10 − 9 = 1 modo. A resposta ´ 38 .2.1 = 13.122.
                                                    e

     a 5                                            5
6) H´ C15 modos de formar o Esporte; depois disso, C10 modos de formar
   o Tupi; finalmente, 1 unico modo de formar o Minas.
                        ´
              e 5      5
   A resposta ´ C15 × C10 × 1 = 756.756.

7) O n´ mero de possibilidades ´ igual ao n´mero obtido no problema
      u                        e           u


                                   8
anterior dividido por 3! = 6, j´ que permutando os nomes dos times a
                                  a
   subdivis˜o continua a mesma. A resposta ´ 756.756/6 = 126.126.
           a                                 e

8) Escolha, sucessivamente, 3 pessoas para formar os 4 grupos de 3; isto
                                             3   3      3       3
   pode ser feito, sucessivamente, de C20 , C17 , C14 e C11 modos. A seguir,
   com as 8 pessoas restantes forme os 2 grupos restantes, o que pode ser
             4     4
   feito de C8 e C4 modos, respectivamente. Fazendo isso, contamos cada
   divis˜o 4!.2! vezes, porque, quando formamos os mesmos grupos de 3
        a
   e os mesmos grupos 4 em outra ordem, contamos como se fosse outra
   divis˜o em grupos.
        a
                 C 3 .C 3 .C 3 C 3 .C 4 .C 4         20!
   A resposta ´ 20 17 14 11 8 4 =
               e                                                  =
                           4!.3!               (3!)4 (4!)2 4!2!
   = 67.897.830.000.
   Outra solu¸˜o:
             ca
   Forme uma fila com as 20 pessoas. Isso automaticamente as divide
   em 4 grupos de 3 e 2 grupo de 4: as 3 primeiras formam um grupo,
   as 3 seguintes formam outro, etc.. H´ 20! modos de formar a fila.
                                              a
   Entretanto, uma mesma divis˜o em grupos corresponde a v´rias filas
                                   a                                  a
   diferentes, o que faz com que, no resultado 20!, cada divis˜o tenha a
   sido contada v´rias vezes. Devemos corrigir nossa contagem dividindo
                     a
   o resultado pelo n´mero de vezes que cada divis˜o foi contada. Tro-
                        u                                a
   cando a ordem dos elementos em cada grupo, o que pode ser feito de
   3!.3!.3!.3!.4!.4! modos, ou a ordem dos grupos, o que pode ser feito de
   4!.2! modos, a divis˜o em grupos n˜o se altera, mas a fila sim. Cada
                         a                a
                                                                        20!
   divis˜o foi, assim, contada (6!)3 .2!.3! vezes e a resposta ´
         a                                                     e      4 (4!)2 4!2!
                                                                                   .
                                                                 (3!)
9) Os advers´rios em cada jogo podem ser escolhidos, sucessivamente, de
               a
     2     2     2   2    2    2
   C12 , C10 , C8 , C6 , C4 e C2 modos. No entanto, assim contamos cada
   poss´ rodada 6! vezes, j´ que contamos diferentes ordens dos jogos
        ıvel                     a
                                                   C 2 .C 2 .C 2 .C 2 .C 2 .C 2
   como se fossem rodadas diferentes. A resposta ´ 12 10 8 6 4 2 =
                                                 e
                                                              6!
    12!
          = 10.395
   26 .6!
   Outra solu¸˜o: ca
   Colocando os 12 times em fila automaticamente formamos os 6 jogos
   da rodada. No entanto, a mesma rodada ´ contada v´rias vezes; os
                                          e          a


                                       9
advers´rios em cada jogo podem ser ordenados de 2 modos, enquanto
             a
    os jogos podem ser ordenados de 6! modos. A resposta ´, portanto,
                                                          e
     12!
           .
    26 .6!
10) a) Para determinar o lugar ocupado pelo n´mero 62.417, devemos con-
                                             u
    tar quantos n´meros est˜o antes dele. Antes dele est˜o 6.0023-380
                 u          a                            a
a 480 dezenas de milhar, ou seja, 4.800.000. A resposta ´ 480 + 4.800
                                                            e
    + 48.000 + 480.000 + 4.800.000 = 5.333.280.
    Outra solu¸˜o:
              ca

    H´ 5! = 120 parcelas na soma. Podemos agrup´-las em 60 pares,
      a                                              a
    juntando a cada n´ mero o que dele se obt´m trocando o 1 com o 7,
                      u                      e
    trocando o 2 com o 6, e conservando a posi¸˜o do 4. Em cada par, a
                                              ca
    soma vale 88.888.
    A resposta ´ 88.888 × 60 = 5.333.280.
               e

11) Devemos inicialmente escolher a ordem em que as mo¸as ficar˜o juntas,
                                                         c      a
    o que pode ser feito de m! maneiras. Em seguida, devemos arrumar
    em fila r + 1 objetos, os r rapazes e o bloco das mo¸as, o que pode ser
                                                       c
    feito de (r + 1)! modos.
    A resposta ´ m!.(r + 1)!.
               e

12) a) A face a receber o n´ mero 1 pode ser escolhida de 6 modos, a do
                           u
    n´ mero 2 de 5 modos, e assim por diante. O n´mero de possibilidades
      u                                          u
    ´ 6.5.4.3.2.1 = 720.
    e
    b) Inicialmente, fazemos de conta que as faces tem cores diferentes.
    Contamos, pelo problema anterior, 720 dados. Como as faces s˜o in-  a
    distingu´ıveis, o mesmo dado foi contado v´rias vezes. Por exemplo,
                                                   a
    pense em um dado que tenha o 6 na face de baixo (face preta) e o 1
    na face de cima (face branca). Ele ´, certamente, diferente de um dado
                                         e
    que tenha o 1 na face de baixo (face preta) e o 6 na face de cima (face
    branca). Mas sendo as faces indistingu´    ıveis, o dado que tem o 6 na
    face de baixo e o 1 na face de cima ´ igual ao dado que tem o 1 na face
                                          e
    de baixo e o 6 na face de cima; este ´, simplesmente, aquele de cabe¸a
                                            e                               c
    para baixo. Esse mesmo dado aparece outra vez com o 1 na face da
    frente e o 6 na face de tr´s, com o 1 na face da esquerda e o 6 na face da
                              a
    direita, etc. Em suma, o mesmo dado foi contado tantas vezes quantas
    s˜o as posi¸˜es de coloc´-lo.
     a          co            a
    O n´ mero de posi¸˜es de colocar um cubo ´ 6×4 = 24, pois h´ 6 modos
        u             co                      e                   a
    de escolher a face de baixo e 4 de escolher, nessa face, o lado que fica
    de frente.
                  720
    A resposta ´e     = 30.
                   24

                                    11
Outra solu¸˜o:
              ca
    Todo dado pode ser imaginado com a face 1 em baixo. Realmente, se
    o 1 n˜o estiver em baixo, ´ poss´ rodar o dado de modo que o 1 v´
          a                    e     ıvel                                  a
    para baixo. Fixado o 1 em baixo, devemos escolher quem ocupar´ a    a
    face oposta a face do 1. Isso pode ser feito de 5 modos. Digamos que
                 `
    tenha sido escolhido o 6. Com o 1 fixo em baixo e o 6 fixo em cima,
    devemos colocar os n´ meros 2, 3, 4 e 5 nas faces laterais. O 2 sempre
                          u
    pode ser imaginado na face da frente. Com efeito, se o 2 n˜o estiver na
                                                               a
    face da frente, uma conveniente rota¸˜o coloc´-lo-´ na face da frente,
                                          ca       a a
    sem tirar o 1 da face de baixo nem o 6 da face de cima. Fixados o 2 na
    frente, o 1 em baixo e o 6 em cima, devemos escolher quem ocupar´ a  a
    face oposta a face do 2. Isso pode ser feito de 3 modos. Digamos que
                 `
    tenha sido escolhido o 4. Agora, devemos colocar o 3 e o 5 nas faces da
    direita e da esquerda. Note que qualquer movimento com o dado ou
    retirar´ o 1 de baixo, ou o 6 de cima, ou o 2 da frente, ou o 4 de tr´s.
            a                                                            a
    Portanto, h´ 2 modos de preencher as faces direita e esquerda com os
                 a
    n´ meros 3 e 5.
      u
    A resposta ´ 5 × 3 × 2 = 30.
               e
    c) Um dado com faces de cores diferentes pode, agora, ser numerado
    de apenas 6.4.2 = 48 modos, j´ que temos 6 faces a escolher para o
                                   a
    n´ mero 1 (isto determina a face do n´mero 6), 4 para o n´mero 2 (o
     u                                   u                    u
    que determina a face do 5) e 2 para o n´mero 3 (que determina a do
                                            u
    4). Mas como as faces s˜o iguais, cada dado ´ contado, como no item
                            a                   e
    anterior, 24 vezes. Logo h´ apenas 48/24 = 2 dados distintos.
                              a
    Outra Solu¸˜o:
              ca
    Como antes, podemos fixar o dado com o n´ mero 1 em baixo. Agora,
                                                u
    no entanto, isto tamb´m fixa o n´ mero 6 na face de cima. Agora, o
                          e          u
    n´ mero 2 pode ser fixado na face da frente (e, portanto, o n´ mero 5 na
      u                                                          u
    de tr´s). Assim, tudo que temos a escolher ´ se a face lateral da direita
         a                                     e
    ´ o 3 ou o 4. Temos, portanto, apenas duas possibilidades.
    e

13) a) O n´ mero de posi¸˜es para um tetraedro regular ´ 4 × 3 = 12, pois
          u              co                            e
    h´ 4 modos de escolher a face de apoio e 4 de escolher, nessa face, o
      a
    lado que fica de frente.
                 4!
    A resposta ´
               e    = 2.
                 12

                                    12
b) O n´ mero de posi¸˜es para um octaedro regular ´ 6 × 4 = 24, pois
           u              co                              e
    h´ 6 modos de escolher o v´rtice de apoio e 4 de escolher, dentre as
      a                          e
    arestas que incidem nesse v´rtice, a que fica de frente.
                                e
                  8!
    A resposta ´
               e      = 1.680.
                  24
    c) O n´ mero de posi¸˜es para um dodecaedro regular ´ 12 × 5 = 60,
           u              co                                 e
    pois h´ 12 modos de escolher a face de apoio e 5 de escolher, nessa face,
          a
    o lado que fica de frente.
                  12!
    A resposta ´
               e      = 7.983.360.
                  60
    d) O n´ mero de posi¸˜es para um icosaedro regular ´ 20 × 3 = 60, pois
           u             co                              e
    h´ 20 modos de escolher a face de apoio e 3 de escolher, nessa face, o
      a
    lado que fica de frente.
                  20!
    A resposta ´
               e      = 40.548.366.802.944.000 ∼ 4.1016 .
                                               =
                  60
14) Temos 1! = 1, que ´ um quadrado perfeito, 1! + 2! = 1 + 2 = 3, que n˜o
                       e                                                  a
    ´ quadrado perfeito, 1! + 2! + 3! = 1 + 2 + 6 = 9, que novamente ´ um
    e                                                                  e
    quadrado perfeito, 1! + 2! + 3! + 4! = 1 + 2 + 6 + 24 = 33, que n˜o ´
                                                                        a e
    quadrado perfeito. Todos os fatoriais seguinte, a partir de 5! terminam
    com zero, j´ que s˜o m´ ltiplos de 5 e 2. Logo, todas as somas da forma
                a     a    u
       n
       1 k! para n ≥ 5 terminam com o algarismo 3 e n˜o s˜o, portanto,
                                                           a a
    quadrados perfeitos. As unicas solu¸˜es s˜o n = 1 e n = 3.
                             ´           co    a
15) Em ESTRELADA as letras A e E aparecem 2 vezes cada e as letras S,
    T, R, L e D aparecem 1 vez cada uma, havendo, portanto, 9 letras na
    palavra.
    Para formar um anagrama, devemos escolher 2 das 9 posi¸˜es paraco
                                                   2
    colocar as letras A, o que pode ser feito de C9 modos, 2 das 7 posi¸˜es
                                                                       co
                                                                 2
    restantes para colocar as letras E, o que pode ser feito de C7 modos, e
    arrumar as letras S, T, R, L e D nas 5 posi¸˜es restantes, o que pode
                                                 co
                                           2   2
    ser feito de 5! modos. A resposta ´ C9 .C7 .5! = 90.720.
                                       e
    Outra solu¸˜o:
              ca
                                                     9!
       u                   e 2,2,1,1,1,1,1 =
    O n´ mero de anagramas ´ P9                               = 90.720.
                                               2!2!1!1!1!1!1!
16) Formar um subconjunto com p elementos significa escolher p dos n
                          e p
    elementos. A resposta ´ Cn .

                                    13
4
17) Basta escolher as provas do primeiro dia, o que pode ser feito de C8 = 70
    modos.

18) O processo de contagem apresentado conta determinadas comiss˜es   o
    mais de uma vez. Isto ocorre porque um homem que participe da
    comiss˜o pode ser inserido de dois modos diferentes: como um dos 3
          a
    homens escolhidos inicialmente, ou como uma das duas pessoas escol-
    hidas posteriormente. O pior ´ que n˜o ´ poss´
                                  e       a e      ıvel ”corrigir” a con-
    tagem dividindo pelo n´mero de vezes que cada comiss˜o ´ contada:
                           u                               a e
    as comiss˜es com 3 homens s˜o contadas apenas uma vez, as que tˆm
             o                  a                                     e
    4 homens s˜
4(4 − 3)
     do poliedro, 12 dos quais s˜o arestas e 6
                                a                       = 12 dos quais s˜o
                                                                        a
                                                   2
     diagonais de faces. A resposta ´ 28 − 12 − 12 = 4.
                                    e
     Outra solu¸˜o:
               ca
     Cada diagonal de um prisma n-agonal une um v´rtice da base “de
                                                       e
     cima” a um v´rtice da base “de baixo”. O v´rtice da base “de cima”
                    e                            e
     pode ser selecionado de n modos; depois disso, o da base ”de baixo”
     pode ser selecionado de n − 3 modos, pois um dos v´rtices da base ”de
                                                       e
     baixo”, se selecionado, daria origem a uma aresta e os dois v´rtices a
                                                                  e
     ele adjacentes nesta base dariam origem a diagonais de faces laterais.
     O n´ mero de diagonais de um prisma n-agonal ´, portanto, n(n − 3).
         u                                         e
     Como o cubo ´ um prisma quadrangular,
                 e
     a resposta ´ 4(4 − 3) = 4.
                e
     e) O prisma hexagonal ´ um poliedro formado por 6 faces quadran-
                              e
     gulares e 2 faces hexagonais e que tem 12 v´rtices e 18 arestas. H´
                                                 e                      a
       2
     C12 = 66 segmentos que unem dois v´rtices do poliedro, 18 dos quais
                                          e
                     4(4 − 3)    6(6 − 3)
     s˜o arestas e 6
      a                       +2          = 30 dos quais s˜o diagonais de
                                                          a
                         2           2
     faces.
     A resposta ´ 66 − 18 − 30 = 18.
                e
     Outra solu¸˜o:
               ca
     O n´ mero de diagonais de um prisma n-agonal ´, como visto em d),
         u                                          e
     n(n − 3). Portanto, o n´ mero de diagonais de um prisma hexagonal ´
                            u                                          e
     6(6 − 3) = 18.
20) Uma fun¸˜o estritamente crescente ´ necessariamente injetiva (se f (a) =
              ca                         e
    f (b), n˜o pode ser a < b, pois, neste caso, f (a) < f (b), o que ´ absurdo;
            a                                                         e
    do mesmo modo, n˜o pode ser a > b, pois, neste caso, f (a) > f (b),
                         a
    o que ´ absurdo; logo, a = b). Logo, seu conjunto de valores ter´
            e                                                                  a
    exatamente m elementos. Para construir uma tal fun¸˜o, devemos, ini-
                                                              ca
                                                                              m
    cialmente, selecionar o conjunto de valores, o que pode ser feito de Cn
    modos.
     Selecionado o conjunto de valores, a fun¸˜o est´ determinada porque
                                             ca     a
     f (1) deve ser igual ao menor elemento do conjunto de valores, f (2)
     deve ser igual ao segundo menor elemento do conjunto de valores, etc.
                              m
     A resposta ´, portanto, Cn .
                e

                                      15
21) Vamos esquecer que a primeira casa do n´mero n˜o pode ser igual a
                                           u       a
    zero. Isso far´ com que contemos a mais e, depois, descontaremos o
                  a
    que foi contado indevidamente.
     a 3
    H´ C7 modos de escolher as casas que ser˜o ocupadas pelo d´
                                                 a                   ıgito 4;
                    a 2
    depois disso, h´ C4 modos de selecionar as casas que ser˜o ocupadas
                                                                a
    pelo d´
          ıgito 8; finalmente, as duas casas restantes podem ser preenchidas
    de 8 × 8 modos (n˜o podemos usar nessas casas os d´
                       a                                  ıgitos 4 e 8).
                        3    2
    A “resposta” seria C7 × C4 × 8 × 8 = 35 × 6 × 64 = 13.440.
    Devemos subtrair os n´meros come¸ados por 0. Se o n´mero come¸a
                           u            c                     u           c
                 3
    por 0, h´ C6 modos de escolher as casas que ser˜o ocupadas pelo d´
             a                                      a                  ıgito
                      a 2
    4; depois disso, h´ C3 modos de selecionar as casas que ser˜o ocupadas
                                                                a
    pelo d´ıgito 8; finalmente, a casa restante pode ser preenchida de 8
    modos (n˜o podemos usar nessa casa os d´
               a                                             a 3    2
                                             ıgitos 4 e 8). H´ C6 ×C3 ×8 =
    20 × 3 × 8 = 480 n´meros come¸ados por 0.
                        u          c
    A resposta ´ 13.440 − 480 = 12.960.
               e
    Outra solu¸˜o:
              ca
    Vamos contar separadamente:
    i) n´ meros que come¸am em 4; ii) n´ meros que come¸am em 8; iii)
        u               c              u               c
    n´ meros que n˜o come¸am nem em 4 nem em 8.
      u           a       c

      i) H´ 1 modo de preencher a primeira casa; depois disso, h´ C6
           a                                                        a 2
         modos de escolher as outras duas casas do n´ mero que tamb´m
                                                      u               e
                                                              a 2
         ser˜o preenchidas com o algarismo 4; depois disso, h´ C4 modos
            a
         de escolher as duas casas que ser˜o ocupadas pelo algarismo 8;
                                           a
         finalmente, as duas casas restantes podem ser preenchidas de 8×8
         modos (n˜o podemos usar nessas casas os d´
                  a                                 ıgitos 4 e 8).
                   2    2
         H´ 1 × C6 × C4 × 8 × 8 = 1 × 15 × 6 × 64 = 5.760 n´ meros do
           a                                                    u
         tipo i).
      ii) H´ 1 modo de preencher a primeira casa; depois disso, h´ 6 modos
            a                                                    a
          de escolher a outra casa do n´ mero que tamb´m ser´ preenchida
                                        u              e      a
                                             a 3
          com o algarismo 8; depois disso, h´ C5 modos de escolher as trˆs
                                                                        e
          casas que ser˜o ocupadas pelo algarismo 4; finalmente, as duas
                        a
          casas restantes podem ser preenchidas de 8 × 8 modos (n˜o pode-
                                                                   a
          mos usar nessas casas os d´
                                    ıgitos 4 e 8).
                        3
          H´ 1 × 6 × C5 × 8 × 8 = 6 × 10 × 64 = 3840 n´meros do tipo ii).
            a                                           u

                                    16
iii) H´ 7 modos de preencher a primeira casa (n˜o podemos usar nem
            a                                        a
                                               3
          4, nem 8, nem 0); depois disso, h´ C6 modos de escolher as trˆs
                                           a                             e
          casas do n´ mero que ser˜o preenchidas com o algarismo 4; depois
                      u           a
                  a 2
          disso, h´ C3 modos de escolher as duas casas que ser˜o ocupadas
                                                               a
          pelo algarismo 8; finalmente, a casa restante pode ser preenchida
          de 8 modos (n˜o podemos usar nessas casas os d´
                         a                                ıgitos 4 e 8).
                    3    2
          H´ 7 × C6 × C3 × 8 = 7 × 20 × 3 × 8 = 3.360 n´ meros do tipo iii).
            a                                           u
          A resposta ´ 5.760 + 3.840 + 3.360 = 12.960.
                       e

22) a) Para formar o subconjunto devemos escolher os p−1 outros elementos
    do subconjunto dentre os n − 1 outros elementos do conjunto.
               e p−1
    A resposta ´ Cn−1 .
    b) Para formar o subconjunto devemos escolher os p elementos do sub-
    conjunto dentre os n − 1 outros elementos do conjunto.
               e p
    A resposta ´ Cn−1 .
    Outra solu¸˜o:
              ca
                                                    p−1
     a p
    H´ Cn p-subconjuntos e o elemento a1 figura em Cn−1 deles. Logo, h´
                                                                     a
     p    p−1
    Cn − Cn−1 subconjuntos nos quais o elemento a1 n˜o figura.
                                                    a
                       p−1
               e p
    A resposta ´ Cn − Cn−1 .
                                                           p   p−1
    Observa¸˜o: As duas solu¸˜es apresentadas mostram que Cn −Cn−1 . =
             ca               co
     p
    Cn−1 . Essa ´ a famosa Rela¸˜o de Stifel.
                e              ca
    c) Para formar o subconjunto devemos escolher os p−2 outros elementos
    do subconjunto dentre os n − 2 outros elementos do conjunto.
               e p−2
    A resposta ´ Cn−2 .
                                 e p
    d) O total de p-subconjuntos ´ Cn . Para formar um subconjunto em que
    nem a1 nem a2 figurem devemos escolher os p elementos do subconjunto
                                                                      p
    dentre os n − 2 outros elementos do conjunto. H´, portanto, Cn−2
                                                        a
    subconjuntos nos quais nem a1 nem a2 figuram. Logo, o n´mero de
                                                                u
    subconjuntos nos quais pelo menos um desses dois elementos figura ´  e
      p    p
    Cn − Cn−2 .
    Outra solu¸˜o:
              ca
      a p−1                                                 a p−1
    H´ Cn−1 p-subconjuntos nos quais o elemento a1 figura e h´ Cn−1
                                                             p−2
    subconjuntos nos quais o elemento a2 figura. H´, tamb´m, Cn−2 p-
                                                 a      e

                                   17
subconjuntos nos quais os elementos a1 e a2 figuram ambos. Ao so-
               p−1   p−1       p−1
    marmos Cn−1 + Cn−1 = 2Cn−1 obtemos o n´ mero de subconjuntos nos
                                             u
    quais pelo menos um dos elementos a1 e a2 figura, mas contamos duas
    vezes aqueles em que a1 e a2 figuram ambos.
                              p−1    p−2
    A resposta ´, portanto, 2Cn−1 − Cn−2 .
               e



    Outra solu¸˜o:
              ca
                              p−2
    H´, como mostrado em c), Cn−2 p-subconjuntos em que os elementos
      a
    a1 e a2 figuram ambos.
      a p−1
    H´ Cn−2 p-subconjuntos em que o elemento a1 figura e o elemento a2
    n˜o figura, pois, para formar um tal subconjunto, basta escolher os
     a
    outros p − 1 elementos do subconjunto dentre os n − 2 elementos do
    conjunto que s˜o diferentes de a1 e de a2 .
                  a
                         p−1
    H´, analogamente, Cn−2 p-subconjuntos em que o elemento a2 figura
      a
    e o elemento a1 n˜o figura. Portanto, o n´mero de p-subconjuntos em
                     a                      u
                                                       p−1    p−2
    que figura pelo menos um desses dois elementos ´ 2Cn−2 + Cn−2 .
                                                  e
                                                      p−1
    e) Como visto na solu¸˜o anterior, a resposta ´ 2Cn−2 .
                         ca                       e
    Outra solu¸˜o:
              ca
                              p−1    p−2
    H´, como visto em d), 2Cn−1 − Cn−2 p-subconjuntos nos quais pelo
      a
                                                                 p−2
    menos um dos elementos a1 e a2 figura. H´, como visto em c), Cn−2
                                              a
    p-subconjuntos em que os elementos a1 e a2 figuram ambos.
                              p−1    p−2    p−2
    A resposta ´, portanto, 2Cn−1 − Cn−2 − Cn−2 =
               e
        p−1     p−2
    = 2Cn−1 − 2Cn−2.
    Outra solu¸˜o:
              ca
                              p−1    p−2
    H´, como visto em d), 2Cn−2 + Cn−2 p-subconjuntos nos quais pelo
      a
                                                                 p−2
    menos um dos elementos a1 e a2 figura. H´, como visto em c), Cn−2
                                              a
    p-subconjuntos em que os elementos a1 e a2 figuram ambos.
                              p−1    p−2    p−2
    A resposta ´, portanto, 2Cn−2 + Cn−2 − Cn−2 =
               e
        p−1
    = 2Cn−2 .

                                     e 5
23) a) Como h´ 32 cartas, a resposta ´ C32 = 201.376.
             a


                                   18
b) H´ 8 modos de escolher o grupo do par propriamente dito (por
     a
                     2
exemplo, valete), C4 modos de escolher os naipes das duas cartas do
                                     3
par (por exemplo, copas e paus), C7 modos de escolher os grupos das
outras trˆs cartas (por exemplo, 10, 8 e rei) e 4 × 4 × 4 = 43 modos de
         e
escolher os naipes dessas trˆs cartas.
                            e
A resposta ´ 8 × C4 × C7 × 43 = 107.520.
           e      2    3

    a 2
c) H´ C8 modos de escolher os grupos das cartas que formar˜o os dois
                                                             a
         2 2
pares, (C4 ) modos de escolher seus naipes, 6 modos de escolher o grupo
da outra carta e 4 modos de escolher seu naipe.
A resposta ´ C8 × (C4 )2 × 6 × 4 = 24.192.
           e 2      2

Observa¸˜o: Um erro muito comum ´ o exposto a seguir.
       ca                       e
                                                   2
H´ 8 modos de escolher o grupo do primeiro par, C4 modos de escolher
  a
os naipes do primeiro par, 7 modos de escolher o grupo do segundo par,
  2
C4 modos de escolher os naipes do segundo par, 6 modos de escolher
o grupo da outra carta e 4 modos de escolher seu naipe. A resposta
                         2      2
ERRADA seria 8 × C4 × 7 × C4 × 6 × 4 = 48.384. A explica¸˜o do ca
ERRO ´ simples: Ao fazermos a inexistente distin¸˜o entre primeiro
        e                                            ca
par e segundo par, contamos pares de valetes e reis como diferentes de
pares de reis e valetes.
A resposta ERRADA pode ser corrigida dividindo-a por 2.
                                               3
d) H´ 8 modos de escolher o grupo da trinca, C4 modos de escolher os
    a
                              2
naipes das cartas da trinca, C7 modos de escolher os grupos das outras
duas cartas e 4×4 = 42 modos de escolher os naipes dessas duas cartas.
A resposta ´ 8 × C4 × C7 × 42 = 10.752.
           e      3    2

e) H´ 8 modos de escolher o grupo do ”four”, 1 modo de escolher os
    a
naipes das quatro cartas do ”four”, 7 modos de escolher o grupo da
outra carta e 4 modos de escolher o naipe dessa carta.
A resposta ´ 8 × 1 × 7 × 4 = 224.
           e
                                               3
f) H´ 8 modos de escolher o grupo da trinca, C4 modos de escolher os
    a
                                                                   2
naipes das cartas da trinca, 7 modos de escolher o grupo do par e C4
modos de escolher os naipes das cartas do par.
                  3        2
A resposta ´ 8 × C4 × 7 × C4 = 1.344.
           e
g) H´ apenas 4 tipos de seq¨ˆncias: 7, 8, 9, 10, valete; 8, 9, 10, valete,
    a                      ue
dama; 9, 10, valete, dama, rei; 10, valete, dama, rei, as. Escolhido o
                                                        ´

                                19
tipo da seq¨ˆncia, haveria 4 × 4 × 4 × 4 × 4 = 45 modos de escolher os
               ue
    naipes das cartas da seq¨ˆncia, mas 4 desses modos n˜o s˜o permitidos:
                            ue                          a a
    todas de ouros, todas de paus, todas de copas e todas de espadas.
    A resposta ´ 4 × [45 − 4] = 4.080.
               e
                                                     5
    h) Os grupos das cartas podem ser escolhidos de C8 − 4 modos e o
    naipe unico, de 4 modos.
          ´
               e 5
    A resposta ´ (C8 − 4) × 4 = 208.
    i) H´ 4 modos de escolher os grupos das cartas e 4 modos de escolher
        a
    o naipe unico.
            ´
    A resposta ´ 4 × 4 = 16.
               e
    j) H´ 4 modos de escolher o naipe unico. A resposta ´ 4.
        a                             ´                 e

24) a) Neste caso f ´ bijetiva e, se #A = #B = n, o n´ mero de fun¸˜es
                    e                                u            co
    f : A → B bijetivas ´ n!, como foi mostrado no exerc´ 4 da se¸˜o
                         e                              ıcio       ca
    2.2.
    b) Neste caso dois elementos de A ter˜o uma mesma imagem em B e
                                         a
    a correspondˆncia entre os demais n − 1 elementos de A e os demais
                e
    n − 1 elementos de B ser´ bijetiva.
                            a
          n+1
    H´
     a            modos de escolher os dois elementos de A, n modos de
            2
    escolher a imagem deles em B e (n − 1)! modos de construir uma
    correspondˆncia bijetiva entre os elementos restantes.
               e
                   n+1                      n(n + 1)!
    A resposta ´e          · n · (n − 1)! =           .
                    2                           2
    c) Neste caso temos as alternativas:

      i) Trˆs elementos de A tˆm a mesma imagem em B e a correspondˆncia
            e                 e                                      e
         entre os demais n − 1 elementos de A e os demais n − 1 elementos
         de B ´ bijetiva.
               e
               n+2
         H´a           modos de escolher os trˆs elementos de A, n modos
                                               e
                 3
         de escolher a imagem deles em B e (n − 1)! modos de construir
         uma correspondˆncia bijetiva entre os elementos restantes.
                          e
               n+2                  n(n + 2)!
         H´a          .n.(n − 1)! =           fun¸˜es desse tipo.
                                                 co
                 3                      6

                                   20
ii) H´ dois pares de elementos de A com imagens idˆnticas em B e a
            a                                              e
          correspondˆncia entre os demais n − 2 elementos de A e os demais
                      e
          n − 2 elementos de B ´ bijetiva.
                                e
                n
          H´a       modos de escolher os dois elementos de
                2
               n+2        n
          B,            ×     modos de escolher suas imagens inversas em A
                  2       2
          e (n − 2)! modos estabelecer a correspondˆncia entre os elementos
                                                   e
          restantes.
            n   n+2   n              n(n − 1)(n + 2)!
    H´
     a        ×     ×   × (n − 2)! =                  fun¸˜es desse
                                                         co
            2    2    2                     8
    tipo.
                   n(n + 2)! n(n − 1)(n + 2)!   n(3n + 1)(n + 2)!
    A resposta ´
               e            +                 =                   .
                       6            8                  24
25) Chamemos de D o conjunto C − C1 .
    H´ quatro tipos de planos:
     a

       i) determinados por trˆs pontos de D;
                             e
      ii) determinados por dois pontos de D e um de C1 ;
     iii) determinados por um ponto de D e dois de C1 ;
     iv) determinados por trˆs pontos de C1 .
                            e

               e 3      2           2
    A resposta ´ C12 + C12 .8 + 12.C8 + 1 = 1.085.


    Outra solu¸˜o:
              ca
    Para determinar um plano, devemos selecionar 3 dos 20 pontos, o que
                       3
    pode ser feito de C20 = 1140 modos. Nessa contagem, o plano que
                                          3
    cont´m os 8 pontos de C1 foi contado C8 = 56 vezes.
        e
    A resposta ´ 1.140 − 56 + 1 = 1.085.
               e
26) Escolhida a ordem de cada casal, o que pode ser feito de 23 modos
    temos que arrumar em fila 4 espa¸os vazios e 3 casais, o que pode ser
                                       c
              4
    feito de C7 modos (escolha dos espa¸os vazios) vezes 3! (coloca¸˜o dos
                                         c                         ca
    3 casais nos 3 lugares restantes).

                                   21
A resposta ´ 23 × C7 × 3! = 1.680.
               e       4


27) Primeiro, colocamos as vogais. Como a letra A aparece 3 vezes e as
    letras U, I e O aparecem 1 vez cada, o n´ mero de modos de dispˆ-las
                                              u                       o
                  6!    720
    e 3,1,1,1 =
    ´ P6              =     = 120. A seguir, colocamos as consoantes em
                  3!     6
    trˆs dos 7 espa¸os antes, entre e depois das vogais. O lugar do P pode
      e              c
    ser qualquer um destes 7 espa¸os, o do R qualquer dos 6 restantes e
                                    c
    o do G qualquer dos 5 restantes. O n´ mero total de possibilidades ´
                                            u                            e
    120.7.6.5 = 25.200.

28) Vamos formar uma fila com os n´ meros 1, 2, . . . n e assinalar com E os
                                     u
    p n´ meros escolhidos e com N os n − p n˜o escolhidos. A condi¸˜o
        u                                     a                         ca
    para que n˜o sejam escolhidos n´meros consecutivos ´ que entre dois E
              a                     u                     e
    haja pelo menos um N. Come¸amos escrevendo os n − p E. A seguir,
                                  c
    devemos escolher, para colocar os E, p dentre os n − p + 1 espa¸os  c
                                                                      p
    situados antes, entre e depois dos N. Isto pode ser feito de Cn−p+1
    modos.

29) Chegam 4 cientistas A, B, C, D. Com as chaves que possuem, abrem
    alguns cadeados, mas n˜o todos. Existe pelo menos um cadeado que
                             a
    eles n˜o conseguem abrir. Na situa¸˜o do n´ mero m´
           a                            ca      u       ınimo de cadeados,
    existe exatamente um cadeado que eles n˜o conseguem abrir. Batize
                                               a
    tal cadeado de ABCD. Portanto, ABCD ´ o cadeado cuja chave n˜o
                                               e                         a
    est´ em poder de A, nem de B, nem de C e nem de D. Qualquer outro
       a
    cientista tem a chave desse cadeado, pois esse cientista e A, B, C e
    D formam um grupo de 5 cientistas e, portanto, nesse grupo algu´m   e
    possui a chave. Como o algu´m n˜o ´ nem A, nem B, nem C e nem D,
                                  e   a e
    deve ser o outro. Analogamente batize os demais cadeados. Verifique
    agora que a correspondˆncia entre cadeados e seus nomes ´ biun´
                            e                                  e    ıvoca,
    isto ´, cadeados diferentes tˆm nomes diferentes (isso porque estamos
         e                       e
    na situa¸˜o do n´ mero m´
              ca     u        ınimo de cadeados) e cadeados de nomes difer-
    entes s˜o diferentes (se X est´ no nome de um cadeado e n˜o est´ no
             a                     a                             a    a
    nome do outro, X tem a chave deste e n˜o tem a chave daquele).
                                             a
    a) O n´ mero m´
          u        ınimo de cadeados ´ igual ao n´mero de nomes de cadea-
                                     e           u
          4
    dos, C1 1 = 330.
    b) Cada cientista X possui as chaves dos cadeados que n˜o possuem X
                                                           a
               4
    no nome, C1 0 = 210.


                                   22
30) Nenhum aluno pode comparecer a mais de trˆs jantares. Com efeito, se
                                                   e
    A1 vai a um jantar com A2 e A3 , ele s´ pode ir a outro jantar com outros
                                          o
    dois estudantes, digamos A4 e A5 e s´ pode ir a um terceiro jantar em
                                           o
    companhia de outros dois, digamos A6 e A7 e n˜o ter´ companhia para
                                                      a    a
    ir a um quarto jantar. Como h´ 21 convites e s˜o 7 estudantes, cada
                                     a                  a
    estudante ter´ que comparecer a exatamente 3 jantares.
                 a
     Se A1 comparece a trˆs jantares, podemos escolher os seus compan-
                            e
     heiros dividindo os outros 6 estudantes em 3 grupos de 2, o que pode
                 C 2 × C4 × 1
                         2
     ser feito de 6            = 15 modos.
                      3!
     Ent˜o, os 3 jantares s˜o, digamos, A1 A2 A3 , A1 A4 A5 , A1 A6 A7 .
         a                 a
     A2 dever´ comparecer a mais dois jantares, nenhum deles em com-
             a
     panhia de A3 , e A3 tamb´m dever´ comparecer a mais dois jantares.
                               e        a
     Portanto, os 4 jantares que faltam s˜o:
                                         a
     A2 , A2 , A3 , A3
     Como A4 deve comparecer a mais dois jantares (A4 n˜o pode compare-
                                                       a
     cer a ambos em companhia de A2 nem a ambos em companhia de A3
     ), esses quatro jantares s˜o:.
                               a
     A2 A4 , A2 , A3 A4 , A3 ;
     A5 tem que comparecer ainda a dois jantares, nenhum deles em com-
     panhia de A4 .
     A2 A4 , A2 A5 , A3 A4 , A3 A5 .
     Agora h´ duas possibilidades:
            a
     A2 A4 A6 , A2 A5 A7 , A3 A4 A7 , A3 A5 A6 e
     A2 A4 A7 , A2 A5 A6 , A3 A4 A6 , A3 A5 A7 .
     H´ portanto 15 × 2 = 30 maneiras de escolher os grupos de convidados.
      a
     Para distribuir os 7 grupos nos 7 dias, h´ 7! alternativas.
                                              a
     A resposta ´ 7! × 30 = 151.200.
                e

31) Os dois primeiros lugares s´ podem ser ocupados por elementos de
                                        o
    {a1 , a2 , · · · a7 } e os dois ultimos por elementos de {a9 , a10 , a11 , a12 }.
                                    ´
                e 2     2
     A resposta ´ C7 × C4 = 126.



                                         23
a m
32) H´ Cm+h × 1 modos de escolher os lugares para os homens. Feito isso,
    s´ h´ 1 modo de formar a fila.
     o a
               e m           m
    A resposta ´ Cm+h × 1 = Cm+h .

33) a) Cada professor fica caracterizado pelas duas bancas a que pertence.
    O n´ mero de professores ´ igual ao n´mero de modos de escolher duas
        u                    e           u
    das oito bancas.
               e 2
    A resposta ´ C8 = 28.
    b) O n´ mero de professores pertencentes a uma banca ´ igual ao n´mero
          u                                              e           u
    de modos de escolher a outra banca a que ele pertence.
    A resposta ´ 7.
               e

34) a) Imagine um quadro em que cada linha ´ a rela¸˜o dos atletas de
                                                   e    ca
    um time. O n´ mero de elementos do quadro ´ o n´ mero de times,
                    u                                e    u
    t, multiplicado pelo tamanho de cada time, k, e ´ tamb´m igual ao
                                                       e      e
    n´ mero de atletas, a, multiplicado pelo n´ mero de times de que cada
      u                                          u
    atleta participa, x.
                           tk
    Logo, ax = tk e x = .
                            a
    b) No mesmo quadro, o n´mero de pares de atletas na mesma linha
                                 u
    ´ igual ao n´mero de linhas, t, multiplicado pelo n´ mero de pares de
    e             u                                     u
                               2
    atletas em uma linha, Ck , e ´ tamb´m igual ao n´mero de pares de
                                   e       e           u
                2
    atletas, Ca , multiplicado pelo n´ mero de times em que cada par de
                                       u
    atletas fica junto, y.
                                 2
              2      2        tCk    tk(k − 1)
    Logo, yCa = tCk e y = 2 =                  .
                              Ca      a(a − 1)
35) A resposta ´ o n´ mero de permuta¸˜es circulares de 4 elementos, ou
                 e  u                co
    seja, 3! = 6.

36) H´ (P C)5 = 4! modos de formar uma roda com as meninas. Depois
      a
    disso, os 5 meninos devem ser postos nos 5 lugares entre as meninas, o
    que pode ser feito de 5! modos. A resposta ´ 4!×5! = 24×120 = 2.880.
                                               e
    ´
37) E mais simples calcular o n´ mero total de rodas e excluir aquelas em
                               u
    que Vera e Isadora ficam juntas. O n´ mero total de rodas ´ P C6 =
                                          u                      e
    5! = 120. Para formar as rodas em que Vera e Isadora ficam juntas, a


                                   24
primeira decis˜o a tomar ´ a ordem em que Vera e Isadora se colocar˜o
                  a          e                                         a
    na roda. H´ 2 possibilidades: Vera-Isadora e Isadora-Vera. Agora tudo
               a
    se passa como se Vera e Isadora fossem uma unica crian¸a. Assim,
                                                    ´          c
    h´ 2(P C)5 = 2.4! = 48 rodas em que Vera e Isadora ficam juntas. A
     a
    resposta ´ 120 − 48 = 72 rodas.
             e

38) Chamando x de 1 + a, y de 1 + b e z de 1 + c, o problema se transforma
    em encontrar todas as solu¸˜es inteiras e n˜o- negativas de (a+1)+(b+
                                 co              a
                                                                4    4
    1) + (c + 1) = 7, ou seja, de a + b + c = 4. A resposta ´ CR3 = C6 = 15.
                                                            e

39) Cada solu¸˜o inteira e n˜o negativa da inequa¸˜o x + y + z ≤ 6 corre-
              ca            a                      ca
    sponde a uma solu¸˜o inteira e n˜o negativa da equa¸˜o x+y+z+f = 6.
                     ca             a                  ca
                 6     6
    Logo, h´ CR4 = C9 = 84 solu¸˜es.
           a                      co

40) Para formar uma caixa, devemos selecionar 20 dentre os 5 tipos, valendo
    repeti¸˜o na escolha. Ou seja, devemos formar solu¸˜es inteiras e n˜o
          ca                                           co                a
    negativas de x1 +x2 +x3 +x4 +x5 = 20, onde xi ´ o n´ mero de bombons
                                                  e    u
                              20     20
    do tipo i. A resposta ´ CR5 = C24 = 10.626.
                          e



    Solu¸˜es dos Exerc´
        co            ıcios da Se¸˜o 5.1
                                 ca

 1) H´ 6×6 = 36 resultados poss´
      a                         ıveis igualmente prov´veis, em 6 dos quais
                                                     a
                                 6     1
    a soma vale 7. A resposta ´
                              e    = .
                                36     6
                                                                        12
 2) Basta escolher os times do primeiro grupo, o que pode ser feito de C24
    modos. Os dois times em quest˜o ficam juntos quando ficam ambos no
                                   a
    primeiro grupo ou ambos no segundo grupo. Em ambos os casos, isto
                                                      10
                       10                           2C22  2.22!.12!    11
    pode ser feito de C22 modos. Logo, a resposta ´ 12 =
                                                  e                  = .
                                                    C24     24!10!     23
    Outra solu¸˜o:
                 ca
    Supondo j´ escolhido o grupo do primeiro time, seus 11 companheiros
              a
                                       11
    de grupo podem ser escolhidos de C23 modos. Dentre os grupos assim
                                                            10
    formados os que tamb´m incluem o segundo time s˜o C22 , j´ que s˜o
                         e                              a       a      a
    formados escolhendo 10 times entre os 22 restantes. Logo, a resposta ´
                                                                         e
      10
    C22    11
      11
         = .
    C23    23

                                    25
3) Usaremos o fato, j´ provado, de que P (A ∪ B) = P (A) + P (B) − P (A ∩
                     a
   B). Temos:
   P (A∪B ∪C) = P ((A∪B) ∪C) = P (A∪B) + P (C) −P ((A∪B) ∩C) =
   P (A) + P (B) − P (A ∩ B) + P (C) − P ((A ∪ B) ∩ C).
   Agora, (A ∪ B) ∩ C = (A ∩ C) ∪ (B ∩ C) e, da´
                                               ı,
   P ((A ∪ B) ∩ C) = P (A ∩ C) + P (B ∩ C) − P ((A ∩ C) ∩ (B ∩ C)) =
   = P (A ∩ C) + P (B ∩ C) − P (A ∩ B ∩ C).
   Substituindo na express˜o anterior, obtemos, finalmente:
                          a
   P (A ∪ B ∪ C) = P (A) + P (B) + P (C)−
   −P (A ∩ B) − P (A ∩ C) − P (B ∪ C) + P (A ∩ B ∩ C).
                                                 2
4) a) Como A ⊂ A ∪ B, temos P (A ∪ B) ≥ P (A) = .
                                                 3
   b) Como A ∩ B e B s˜o disjuntos e A ∩ B ∪ B = A ∪ B, temos
                      a
   P (A ∩ B) + P (B) = P (A ∪ B) e, portanto,
                                                 4
   P (A ∩ B) = P (A ∪ B) − P (B) = P (A ∪ B) − .
                                                 9
                           2
   Mas, do item anterior, ≤ P (A ∪ B) ≤ 1. Da´  ı,
                           3
   2 4                        4          2               5
     − ≤ P (A ∩ B) ≤ 1 − , ou seja, ≤ P (A ∩ B) ≤ .
   3 9                        9          9               9
   c) Observe que P (A ∩ B) + P (A ∩ B) = P (A), j´ que o conjunto da
                                                    a
   direita ´ a uni˜o disjunta dos da esquerda. Da´ P (A ∩ B) = P (A) −
           e      a                              ı,
                2                     2               5       2 5
   P (A ∩ B) = − P (A ∩ B). Como ≤ P (A ∩ B) ≤ , temos − ≤
                3                     9               9       3 9
                 2 2            1               4
   P (A ∩ B) ≤ − , ou seja, ≤ P (A ∩ B) ≤ .
                 3 9            9               9
5) a) O n´ mero de casos poss´
          u                    ıveis ´ 65 , pois h´ 6 resultados para cada um
                                     e            a
                                                           2
   dos 5 dados. O n´ mero de casos favor´veis ´ 6.C5 .5.4.3 = 3600, pois
                     u                        a      e
   h´ 6 modos de escolher o tipo do par (pode ser de 1, de 2, ..., de 6)
    a
       a 2
   e h´ C5 modos de escolher os dois dados que formar˜o o par; quanto
                                                              a
   aos outros dados, o resultado do primeiro deles pode ser escolhido de
   5 modos distintos (deve ser diferente do resultado dos dois primeiros
   dados), o do segundo pode ser escolhido de 4 modos distintos (deve ser
   diferente dos anteriores) e, o do terceiro, de 4 modos diferentes.

                                    26
3.600    25
Logo, P (A2) =           =      ≡ 0, 463.
                     65     54
b) O n´ mero de casos poss´
         u                    ıveis ´ 65 , pois h´ 6 resultados para cada um
                                    e            a
                     u                      a      e 2 2 2
dos 5 dados. O n´ mero de casos favor´veis ´ C6 .C5 .C3 .4 = 1800, pois
       2
h´ C6 modos de escolher os tipos dos pares (podem ser de 1 e 2, de 1 e
 a
                     a 2
3, ..., de 5 e 6), h´ C5 modos de escolher os dois dados que formar˜o o  a
                 2
par menor e C3 modos de escolher os dados que formar˜o o par maior.
                                                              a
Para o dado restante, que deve ter resultado diferente do dos demais
dados, h´ 4 resultados poss´
           a                   ıveis.
                   180    25
Logo, P (A3) = 5 =             ≡ 0, 231.
                    6     108
Observa¸˜o: ca
Um erro bastante comum ´ contar os casos favor´veis em dobro, racioci-
                         e                      a
nando do modo seguinte: H´ 6 modos de escolher o tipo do primeiro
                            a
                                                      2
par, 5 modos de esco lher o tipo do segundo par, C5 modos de escol-
                                                     2
her os dois dados que formar˜o o primeiro par, C3 modos de escol-
                              a
her os dados que formar˜o o segundo par e h´ 4 modos de escolher
                         a                      a
o resultado do dado restante. Logo, o n´ mero de casos favor´veis ´
                                          u                      a      e
      2  2
6.5.C5 .C3 .4 = 3.600.
´
E claro que o erro prov´m da distin¸˜o artificial entre o primeiro e o se-
                       e           ca
gundo par, que faz com que um par de 2 e um par de 5 seja considerado
diferente de um par de 5 e um par de 2.
      u                   ıveis ´ 65 , pois h´ 6 resultados para cada um
c) O n´ mero de casos poss´      e            a
                                                     3
dos 5 dados. O n´ mero de casos favor´veis ´ 6.C5 .5.4 = 1200, pois h´
                 u                       a      e                        a
6 modos de escolher o tipo da trinca (pode ser de 1, de 2, ..., de 6) e h´
                                                                         a
  3
C5 modos de escolher os trˆs dados que ter˜o resultados iguais; quanto
                           e                  a
aos outros dados, o resultado do primeiro deles pode ser escolhido de
5 modos distintos (deve ser diferente do resultado dos trˆs primeiros
                                                              e
dados) e, o do segundo, de 4 modos distintos (deve ser diferente dos
anteriores).
               1.200     25
Logo, P (A4) =     5
                      =       ≡ 0, 154.
                 6       162
d) O n´ mero de casos poss´
       u                   ıveis ´ 65 , pois h´ 6 resultados para cada um
                                 e            a
                                                      4
dos 5 dados. O n´ mero de casos favor´veis ´ 6.C5 .5 = 150, pois h´ 6
                 u                        a      e                     a
modos de escolher o tipo da quadra (pode ser de 1, de 2, ..., de 6) e
 a 4
h´ C5 modos de escolher os quatro dados que ter˜o resultados iguais;
                                                      a


                                 27
quanto ao dado restante, seu resultado pode ser escolhido de 5 modos
   distintos (deve ser diferente do resultado dos quatro primeiros dados).
                   150       25
   Logo, P (A6) = 5 =             ≡ 0, 019.
                    6      1.296
   e) O n´ mero de casos poss´
           u                   ıveis ´ 65 e o n´ mero de casos favor´veis ´ 6.
                                     e          u                    a     e
                    6       1
   Logo, P (A7) = 5 =            ≡ 0, 0008.
                   6      1.296
   f) O n´ mero de casos poss´
          u                    ıveis ´ 65 , pois h´ 6 resultados para cada um
                                     e             a
   dos 5 dados. H´ dois tipos poss´
                   a                  ıveis de seq¨ˆncias: a m´
                                                     ue         ınima (12345)
   e a m´xima (23456). A m´
          a                    ınima pode ser formada de 5! = 120 modos
   distintos, pois h´ 5 modos de escolher o dado cujo resultado ´ 1, 4
                     a                                                  e
   modos de escolher o dado cujo resultado ´ 2, etc. H´, analogamente,
                                                  e           a
   120 modos de formar a seq¨ˆncia m´xima.
                                ue         a
                        240      5
   Portanto, P (A8) = 5 =            ≡ 0, 031.
                         6      162
   g) O n´ mero de casos poss´
            u                    ıveis ´ 65 , pois h´ 6 resultados para cada
                                       e              a
                                                             3
   um dos 5 dados. O n´ mero de casos favor´veis ´ 6.C5 .5 = 300, pois h´
                         u                        a     e                    a
   6 modos de escolher o tipo da trinca (pode ser de 1, de 2, ..., de 6) e
    a 3
   h´ C5 modos de escolher os trˆs dados que formar˜o a trinca; quanto
                                    e                      a
   aos outros dados, h´ 5 modos distintos de escolher o resultado comum
                       a
   deles.
                   300      25
   Logo, P (A5) = 5 =           ≡ 0, 039.
                    6      648
6) Numeremos os v´rtices do pol´
                    e             ıgono de 0 a 2n. Imagine 0 como o v´rtice
                                                                      e
   mais alto, os v´rtices de 1 a n do lado direito e os v´rtices de n + 1 a
                  e                                      e
   2n do lado esquerdo. Podemos pensar que todos os triˆngulos tˆm 0
                                                             a         e
                                        2n(2n − 1)
                   e          a 2
   como um dos v´rtices. H´ C2n =                   = n(2n − 1) modos de
                                              2
   selecionar os outros dois v´rtices do triˆngulo.
                              e             a
   Para contar o n´ mero de triˆngulos que contˆm o centro da circun-
                     u            a                e
   ferˆncia circunscrita em seu interior, observe inicialmente que a reta
      e
   que cont´m o v´rtice i(1 ≤ i ≤ n) e o centro da circunferˆncia corta
            e       e                                         e
   novamente o pol´  ıgono no ponto m´dio do segmento determinado pelos
                                       e
   v´rtices i + n e i + n + 1 (v´rtice 2n + 1 = v´rtice 0).
    e                           e                e
   Um triˆngulo que contenha em seu interior o centro da circunferˆncia
         a                                                        e
   ser´ necessariamente formado por um v´rtice do lado direito e um
      a                                   e

                                    28
v´rtice do lado esquerdo. Se o v´rtice do lado direito for o v´rtice
    e                                     e                            e
   1, o do lado esquerdo s´ poder´ ser o v´rtice n + 1 (1 possibilidade);
                               o      a        e
   se for o v´rtice 2, poder´ ser qualquer dos v´rtices de n + 1 a n + 2 (2
             e                 a                  e
   possibilidades); ...; se for o v´rtice n, poder´ ser qualquer dos v´rtices
                                   e              a                   e
   de n + 1 a 2n (n possibilidades). O n´ mero de casos favor´veis ´
                                                u                    a      e
                       n(n + 1)
   1+2+ ···+n =                  .
                            2
                    n+1
   A resposta ´e              .
                  2(2n − 1)
7) Imagine o resultado do sorteio como uma fila de 12 lugares: o primeiro
   lugar corresponde a primeira pessoa sorteada para o primeiro grupo; o
                      `
   segundo, a segunda pessoa sorteada para o segundo grupo; ...; o ultimo,
            `                                                       ´
   a
   ` quarta pessoa sorteada para o terceiro grupo. Colocada a primeira
   pessoa, h´ 11 posi¸˜es para a segunda, em 3 das quais ela fica no mesmo
            a        co
   grupo da primeira.
                  3
   A resposta ´ .
               e
                 11
8) H´ 12 possibilidades para o signo de cada pessoa, para um total de 124
     a
   possibilidades. Para que n˜o haja coincidˆncias de signos, o signo da
                              a              e
   primeira pessoa pode ser escolhido de 12 modos, o da segunda de 11,
   o da terceira de 10 e o da quarta de 9, para um total de 12.11.10.9
   modos. Assim, a probabilidade de que n˜o haja coincidˆncia de signos
                                           a              e
     12.11.10.9    55
   ´
   e             =    e a probabilidade de que n˜o haja coincidˆncias ´
                                                  a              e      e
        124        96
       55     41
   1−      = .
       96     96
     a 4
9) H´ C10 modos de retirar 4 p´s de sapatos. Para retirar 4 p´s, havendo
                                e                              e
   nesses 4 p´s exatamente 1 par de sapatos, devemos inicialmente sele-
             e
   cionar 1 par (o que pode ser feito de 5 modos) e depois selecionar 2 p´s
                                                                         e
   vindo de pares diferentes dentre os 4 pares que ainda est˜o no arm´rio.
                                                            a         a
                                                                          4
   Para isso devemos escolher os pares de onde vir˜o esses sapatos (C2
                                                      a
   modos) e, em cada par escolhido, decidir se retiraremos o p´ direito ou
                                                               e
                    2
   o p´ esquerdo (2 = 4 modos).
      e
                     2
                  5.C4 .4  4
   A resposta ´
              e      4
                          = .
                   C10     7


                                   29
10) Para distribuir os sorvetes, devemos escolher as pessoas que receber˜oa
                          5
    sorvetes de creme (C10 modos) e dar sorvetes de chocolate as demais
                                                                  `
    (1 modo). Para distribuir os sorvetes, respeitando as preferˆncias,
                                                                      e
    come¸amos dando sorvetes de creme aos que gostam de creme e de
         c
    chocolate aos que gostam de chocolate (1 modo). Em seguida, deve-
    mos distribuir 2 sorvetes de creme e 3 sorvetes de chocolate a 5 pessoas
    que n˜o tˆm preferˆncias; para isso, devemos escolher as 2 pessoas que
          a e           e
                                    2
    receber˜o sorvetes de creme (C5 modos) e dar sorvetes de chocolate as
           a                                                              `
    restantes (1 modo).
                     2
                   C5     5
    A resposta ´
               e    5
                       =     .
                   C10   126
11) As pe¸as do domin´ s˜o formadas por dois, n˜o necessariamente dis-
            c          o a                         a
                                                     2     2
    tintos, dos n´ meros 0, 1, 2, 3, 4, 5 e 6. H´ CR7 = C8 = 28 pe¸as e
                 u                              a                    c
          2
    h´ C28 modos de selecionar duas pe¸as de um domin´. Para selecionar
      a                                  c               o
    duas pe¸as com um n´ mero comum, deve-se primeiramente selecionar
              c           u
    o n´ mero comum (7 possibilidades) e, depois, selecionar 2 das 7 pe¸as
        u                                                              c
                                         2
    que contˆm esse n´ mero comum (C7 possibilidades).
               e      u
                       2
                   7C7    7
    A resposta ´
               e     2
                         = .
                   C28    18
12) O n´ mero de sorteios poss´
       u                            e 5
                              ıveis ´ C80 .
    a) O apostador acerta 3 dezenas quando s˜o sorteadas 3 das 8 dezenas
                                            a
    em que apostou e 2 das 72 em que n˜o apostou. Tais sorteios podem
                                        a
                      3   2
    ser efetuados de C8 .C72 modos.
                   3    2
                  C8 .C72                                 1
    A resposta ´
               e      5
                          (que ´ aproximadamente igual a
                               e                             ).
                   C80                                   168
    b) O apostador acerta 4 dezenas quando s˜o sorteadas 4 das 8 dezenas
                                            a
    em que apostou e 1 das 72 em que n˜o apostou. Tais sorteios podem
                                         a
                        4  1
    ser efetuados de C8 .C72 modos.
                  4   1
                C8 .C72                                  1
    A resposta ´
               e    5
                        (que ´ aproximadamente igual a
                             e                               ).
                  C80                                   4770
    c) O apostador acerta 5 dezenas quando s˜o sorteadas 5 das 8 dezenas
                                            a
                                                           5
    em que apostou. Tais sorteios podem ser efetuados de C8 modos.
                     5
                   C8       1
    A resposta ´
               e    5
                       =         .
                   C80   429.286

                                     30
13) Colocada a primeira pessoa na roda, h´ n − 1 posi¸˜es poss´
                                          a           co       ıveis para
    a segunda pessoa, das quais 2 s˜o favor´veis a que ela fique junto da
                                   a        a
    primeira pessoa.
                   2
    A resposta ´
               e      .
                 n−1
14) a) H´ n posi¸˜es igualmente prov´veis que a chave “certa” poderia
         a        co                   a
    ocupar: ser a primeira a ser testada, a segunda,..., a ultima. A proba-
                                                           ´
    bilidade de ela ocupar a k-´sima posi¸˜o ´ 1/n.
                               e          ca e
    Outra solu¸˜o:
              ca
    H´ n! maneiras de ordenar as chaves a serem tentadas. Para formar as
      a
    ordena¸˜es que tem a chave certa na k-´sima posi¸˜o, devemos colocar
            co                             e          ca
    as n − 1 chaves restantes nas n − 1 posi¸˜es restantes, o que pode ser
                                             co
    feito de (n − 1)! modos. Logo, a probabilidade de que a chave certa
                           (n − 1)!   1
    esteja na posi¸˜o k ´
                   ca    e          = .
                              n!     n
    b) As primeiras k tentativas podem ser feitas de nk modos (cada chave
    pode ser escolhida de n modos, j´ que chaves correspondentes a tenta-
                                     a
    tivas frustradas n˜o s˜o descartadas). Para que se acerte na k- ´sima
                       a a                                          e
    tentativa, as primeiras k − 1 chaves devem ser incorretas (portanto,
    podem ser escolhidas de (n − 1)k−1 modos) e a de ordem k deve ser
    a correta (1 modo). Logo, a probabilidade de se acertar na k-´sima
                                                                    e
                        k−1
                 (n − 1)
    tentativa ´e            .
                     nk
        a 4
15) a) H´ C12 = 495 modos de selecionar as 4 vagas que n˜o ser˜o ocupadas
                                                         a      a
    e 9 modos de escolher 4 vagas consecutivas (1 2 3 4, 2 3 4 5, ..., 9 10
    11 12).
                   9     1
    A resposta ´e      = .
                  495   55
            4
    b) H´ C12 = 495 modos de selecionar as 4 vagas que n˜o ser˜o ocupadas.
        a                                               a      a
    Para contar o n´mero de possibilidades em que n˜o h´ vagas vazias
                     u                                 a a
    adjacentes, devemos escolher 4 dos 9 espa¸os existentes antes, entre e
                                              c
                                                                   4
    depois dos carros para ficarem vazios. Isto pode ser feito de C9 modos.
                                                                  126   14
    Logo a probabilidade de que n˜o haja vagas consecutivas ´ ´
                                 a                            ee      = .
                                                                  495   55
16) P (A ∩ B ∩ C) = 0, pois A ∩ B ∩ C ⊂ A ∩ C e P (A ∩ C) = 0.

                                   31
a) P (A ∪ B ∪ C) = P (A) + P (B) + P (C) − P (A ∩ B) −
    − P (A ∩ C) − P (B ∩ C) + P (A ∩ B ∩ C) =
    = 0, 4 + 0, 5 + 0, 3 − 0, 3 − 0 − 0, 1 + 0 = 0, 8.

    b) P [A − (B ∪ C)] = P (A) − P [A ∩ (B ∪ C)] =
    = P (A) − P [(A ∩ B) ∪ (A ∩ C)] =
    = P (A) − P (A ∩ B) − P (A ∩ C) + P [(A ∩ B) ∩ (A ∩ C)] =
    = P (A) − P (A ∩ B) − P (A ∩ C) + P (A ∩ B ∩ C) =
    = 0, 4 − 0, 3 − 0 + 0 = 0, 1.

    c) P [A ∩ (B ∪ C)] = P [(A ∩ B) ∪ (A ∩ C)] =
    = P (A ∩ B) + P (A ∩ C) − P [(A ∩ B) ∩ (A ∩ C)] =
    = P (A ∩ B) + P (A ∩ C) − P (A ∩ B ∩ C) = 0, 3 + 0 − 0 = 0, 3.

    d) P [(A ∩ B) ∪ C] = P (A ∩ B) + P (C) − P (A ∩ B ∩ C) =
    = 0, 3 + 0, 3 − 0 = 0, 6

18) a) A resposta, naturalmente, ´ 1/2, j´ que, de todos os pares de n´ meros
                                 e       a                            u
    distintos de 1 a 100, em exatamente a metade o primeiro n´mero ´ u      e
    maior do que o segundo.
    b) O n´ mero total de poss´
           u                    ıveis extra¸˜es ´ 100 × 100 = 10.000, j´ que
                                           co e                        a
    o bilhete de cada uma das mo¸as pode ser escolhido de 100 modos. Em
                                   c
    100 destas poss´ıveis extra¸˜es os dois n´ meros s˜o iguais e em metade
                               co            u        a
    das restantes, ou seja, em 9900/2 = 4950 delas, o primeiro n´mero ´
                                                                    u      e
    maior do que o segundo. Logo, a probabilidade de o n´mero de Laura
                                                            u
                                      4950
    ser maior do que o de Telma ´  e        = 0, 495.
                                      10000
19) a) S˜o feitos 5 testes quando uma das quatro primeiras pilhas testadas
         a
    est´ descarregada, o mesmo ocorrendo com a quinta a ser testada. A
       a
    primeira pilha a ser testada pode ser escolhida de 10 modos, a segunda
    de 9, e assim por diante, para um total de 10.9.8.7.6 modos poss´ ıveis
    para escolher as 5 primeiras pilhas a serem testadas. Para formar uma
    sequˆncia de teste em que a segunda defeituosa ´ detectada na 5a ten-
         e                                           e
    tativa, devemos escolher a pilha defeituosa que aparece na 5a posi¸˜o
                                                                       ca
    (2 modos), a posi¸˜o da outra defeituosa (4 modos) e, finalmente, as
                       ca

                                      32
pilhas n˜o defeituosas para as demais posi¸˜es (8.7.6 modos). Logo, a
            a                                  co
                              2.4.8.7.6    4
   probabilidade pedida ´ e             = .
                             10.9.8.7.6   45
   b) S˜o efetuados at´ 5 testes quando as pilhas defeituosas aparecem
         a              e
   nas 5 primeiras tentativas. Como visto no item anterior, h´ 10.9.8.7.6
                                                                  a
   modos de se fazer estas tentativas. Para formar aquelas em que as
   duas defeituosas est˜o entre as testadas devemos escolher a posi¸˜o
                        a                                                 ca
   da primeira pilha defeituosa (5 modos), a da segunda (4 modos) e,
   finalmente, as pilhas n˜o defeituosas para as outras tentativas (8.7.6
                            a
                                                                 5.4.8.7.6
   modos). A probabilidade de que sejam feitos at´ 5 testes ´
                                                   e          e             =
                                                                 10.9.8.7.6
   2                                                     2     7
      e, portanto, a probabilidade pedida ´ igual a 1 − = .
                                             e
   9                                                     9     9
   c) Para que sejam feitos menos de 4 testes, as duas pilhas defeituosas
   devem aparecer nos primeiros 4 testes. O n´ mero total de escolhas
                                                  u
   para os 4 primeiros testes ´ 10.9.8.7. Para formar uma sequˆncia de
                                 e                                    e
   teste em que as duas defeituosas aparecem nestas 4 tentativas, devemos
   escolher a posi¸˜o da primeira pilha defeituosa (4 modos), a da segunda
                  ca
   (3 modos) e, finalmente, as pilhas n˜o defeituosas para as duas outras
                                         a
                                                      4.3.8.7       2
   posi¸˜es (8.7 modos). A probabilidade pedida ´
        co                                         e           = .
                                                      10.9.8.7     15


   Solu¸˜es dos Exerc´
       co            ıcios da Se¸˜o 5.2
                                ca

1) Sejam X e Y os resultados do primeiro e segundo lan¸amentos, respec-
                                                      c
   tivamente. P (X = 3 | X + Y = 7) =
      P (X = 3, X + Y = 7)    1/6 · 1/6   1
   =                       =            = .
          P (X + Y = 7)         6/36      6
   Outra solu¸˜o:
               ca
              e    a             ıveis igualmente prov´veis: X = 1, Y =
   Se a soma ´ 2, h´ 6 casos poss´                    a
   6; X = 2, Y = 5; X = 3, Y = 4; X = 4, Y = 3; X = 5, Y = 2; X =
   6, Y = 1. Dos seis casos, somente X = 3, Y = 3 ´ favor´vel. A
                                                         e     a
              1
   resposta ´ .
            e
              6
                             P (n˜o sabe e acerta)
                                 a
2) P (n˜o sabe | acerta) =
       a                                           =
                                   P (acerta)

                                      33
Mat em combinatoria sol vol2 cap4
Mat em combinatoria sol vol2 cap4
Mat em combinatoria sol vol2 cap4
Mat em combinatoria sol vol2 cap4
Mat em combinatoria sol vol2 cap4
Mat em combinatoria sol vol2 cap4
Mat em combinatoria sol vol2 cap4
Mat em combinatoria sol vol2 cap4

More Related Content

What's hot

Brincando e Aprendendo com a Matemática Equação
Brincando e Aprendendo com a Matemática EquaçãoBrincando e Aprendendo com a Matemática Equação
Brincando e Aprendendo com a Matemática Equação
ivanetelimaseixas
 
Apostila bnb.2014 matematica_dudan
Apostila bnb.2014 matematica_dudanApostila bnb.2014 matematica_dudan
Apostila bnb.2014 matematica_dudan
Eliene Meira
 
Multiplicação e divisão
Multiplicação e divisãoMultiplicação e divisão
Multiplicação e divisão
Maria Cristina
 
2010 volume2 cadernodoaluno_matematica_ensinofundamentalii_8aserie_gabarito
2010 volume2 cadernodoaluno_matematica_ensinofundamentalii_8aserie_gabarito2010 volume2 cadernodoaluno_matematica_ensinofundamentalii_8aserie_gabarito
2010 volume2 cadernodoaluno_matematica_ensinofundamentalii_8aserie_gabarito
profzwipp
 
Equações literais
Equações literaisEquações literais
Equações literais
aldaalves
 
03 operaes algbricas
03 operaes algbricas03 operaes algbricas
03 operaes algbricas
resolvidos
 

What's hot (17)

Fatoração
FatoraçãoFatoração
Fatoração
 
Apostila nivelamento
Apostila nivelamentoApostila nivelamento
Apostila nivelamento
 
Nm
NmNm
Nm
 
Sistemas de equações so 1º grau apresentação
Sistemas de equações so 1º grau apresentaçãoSistemas de equações so 1º grau apresentação
Sistemas de equações so 1º grau apresentação
 
Sequencias e mf 2016
Sequencias e mf 2016Sequencias e mf 2016
Sequencias e mf 2016
 
Brincando e Aprendendo com a Matemática Equação
Brincando e Aprendendo com a Matemática EquaçãoBrincando e Aprendendo com a Matemática Equação
Brincando e Aprendendo com a Matemática Equação
 
Apostila bnb.2014 matematica_dudan
Apostila bnb.2014 matematica_dudanApostila bnb.2014 matematica_dudan
Apostila bnb.2014 matematica_dudan
 
Módulo 01 - 9 ano- Matemática / Ens.Fundamental
Módulo 01 - 9 ano- Matemática  / Ens.FundamentalMódulo 01 - 9 ano- Matemática  / Ens.Fundamental
Módulo 01 - 9 ano- Matemática / Ens.Fundamental
 
Ap matemática m1
Ap matemática m1Ap matemática m1
Ap matemática m1
 
Multiplicação e divisão
Multiplicação e divisãoMultiplicação e divisão
Multiplicação e divisão
 
2010 volume2 cadernodoaluno_matematica_ensinofundamentalii_8aserie_gabarito
2010 volume2 cadernodoaluno_matematica_ensinofundamentalii_8aserie_gabarito2010 volume2 cadernodoaluno_matematica_ensinofundamentalii_8aserie_gabarito
2010 volume2 cadernodoaluno_matematica_ensinofundamentalii_8aserie_gabarito
 
Curso dos Correios Matemática
Curso dos Correios  MatemáticaCurso dos Correios  Matemática
Curso dos Correios Matemática
 
Equações literais
Equações literaisEquações literais
Equações literais
 
Análise combinatória
Análise  combinatóriaAnálise  combinatória
Análise combinatória
 
Aplicações da Congruência Linear
Aplicações da Congruência LinearAplicações da Congruência Linear
Aplicações da Congruência Linear
 
03 operaes algbricas
03 operaes algbricas03 operaes algbricas
03 operaes algbricas
 
Apostila Professor Linhares
Apostila Professor LinharesApostila Professor Linhares
Apostila Professor Linhares
 

Similar to Mat em combinatoria sol vol2 cap4

Implementação modulo3
Implementação modulo3Implementação modulo3
Implementação modulo3
inechidias
 
Implementação modulo3
Implementação modulo3Implementação modulo3
Implementação modulo3
inechidias
 
Implementação modulo3
Implementação modulo3Implementação modulo3
Implementação modulo3
inechidias
 
Implementação modulo3
Implementação modulo3Implementação modulo3
Implementação modulo3
inechidias
 
Analise combinatoria 1
Analise combinatoria 1Analise combinatoria 1
Analise combinatoria 1
SEDUC-PA
 
Implementação modulo3
Implementação modulo3Implementação modulo3
Implementação modulo3
inechidias
 
prof.Calazans(Mat. e suas Tecnologias)-Simulado comentado 03
prof.Calazans(Mat. e suas Tecnologias)-Simulado comentado 03prof.Calazans(Mat. e suas Tecnologias)-Simulado comentado 03
prof.Calazans(Mat. e suas Tecnologias)-Simulado comentado 03
ProfCalazans
 
Sf2n1 2010
Sf2n1 2010Sf2n1 2010
Sf2n1 2010
cavip
 
Correios simulado de matemática
Correios   simulado de matemáticaCorreios   simulado de matemática
Correios simulado de matemática
Rodrigo Lucas
 
Sf2n2 2011
Sf2n2 2011Sf2n2 2011
Sf2n2 2011
cavip
 
2010 volume3 cadernodoaluno_matematica_ensinofundamentalii_7aserie_gabarito
2010 volume3 cadernodoaluno_matematica_ensinofundamentalii_7aserie_gabarito2010 volume3 cadernodoaluno_matematica_ensinofundamentalii_7aserie_gabarito
2010 volume3 cadernodoaluno_matematica_ensinofundamentalii_7aserie_gabarito
profzwipp
 
Ft 12-probabilidades-revisao
Ft 12-probabilidades-revisaoFt 12-probabilidades-revisao
Ft 12-probabilidades-revisao
Laurinda Barros
 

Similar to Mat em combinatoria sol vol2 cap4 (20)

Implementação modulo3
Implementação modulo3Implementação modulo3
Implementação modulo3
 
Implementação modulo3
Implementação modulo3Implementação modulo3
Implementação modulo3
 
Implementação modulo3
Implementação modulo3Implementação modulo3
Implementação modulo3
 
Implementação modulo3
Implementação modulo3Implementação modulo3
Implementação modulo3
 
Implementação modulo3
Implementação modulo3Implementação modulo3
Implementação modulo3
 
Implementação modulo3
Implementação modulo3Implementação modulo3
Implementação modulo3
 
Implementação modulo3
Implementação modulo3Implementação modulo3
Implementação modulo3
 
Implementação modulo3
Implementação modulo3Implementação modulo3
Implementação modulo3
 
Analise combinatoria 1
Analise combinatoria 1Analise combinatoria 1
Analise combinatoria 1
 
Implementação modulo3
Implementação modulo3Implementação modulo3
Implementação modulo3
 
Apostila professor Linhares
Apostila professor LinharesApostila professor Linhares
Apostila professor Linhares
 
prof.Calazans(Mat. e suas Tecnologias)-Simulado comentado 03
prof.Calazans(Mat. e suas Tecnologias)-Simulado comentado 03prof.Calazans(Mat. e suas Tecnologias)-Simulado comentado 03
prof.Calazans(Mat. e suas Tecnologias)-Simulado comentado 03
 
Sf2n1 2010
Sf2n1 2010Sf2n1 2010
Sf2n1 2010
 
Correios simulado de matemática
Correios   simulado de matemáticaCorreios   simulado de matemática
Correios simulado de matemática
 
Raciocínio lógico
Raciocínio lógicoRaciocínio lógico
Raciocínio lógico
 
Soluções pré
Soluções préSoluções pré
Soluções pré
 
Sf2n2 2011
Sf2n2 2011Sf2n2 2011
Sf2n2 2011
 
2010 volume3 cadernodoaluno_matematica_ensinofundamentalii_7aserie_gabarito
2010 volume3 cadernodoaluno_matematica_ensinofundamentalii_7aserie_gabarito2010 volume3 cadernodoaluno_matematica_ensinofundamentalii_7aserie_gabarito
2010 volume3 cadernodoaluno_matematica_ensinofundamentalii_7aserie_gabarito
 
Proposta de-correccao-do-teste-intermedio-9-ano7-de-fevereiro-de-2011-v1
Proposta de-correccao-do-teste-intermedio-9-ano7-de-fevereiro-de-2011-v1Proposta de-correccao-do-teste-intermedio-9-ano7-de-fevereiro-de-2011-v1
Proposta de-correccao-do-teste-intermedio-9-ano7-de-fevereiro-de-2011-v1
 
Ft 12-probabilidades-revisao
Ft 12-probabilidades-revisaoFt 12-probabilidades-revisao
Ft 12-probabilidades-revisao
 

More from trigono_metrico

Pro cefet fasciculo 03 resolução comentada
Pro cefet fasciculo 03 resolução comentadaPro cefet fasciculo 03 resolução comentada
Pro cefet fasciculo 03 resolução comentada
trigono_metrico
 
Pro cefet fasciculo 04 resolução comentada
Pro cefet fasciculo 04 resolução comentadaPro cefet fasciculo 04 resolução comentada
Pro cefet fasciculo 04 resolução comentada
trigono_metrico
 
Ap geometria plana resolvidos
Ap geometria plana resolvidosAp geometria plana resolvidos
Ap geometria plana resolvidos
trigono_metrico
 
Dfato vestibular fasciculo 3
Dfato vestibular fasciculo  3Dfato vestibular fasciculo  3
Dfato vestibular fasciculo 3
trigono_metrico
 
Ap geometria analitica resolvidos
Ap geometria analitica resolvidosAp geometria analitica resolvidos
Ap geometria analitica resolvidos
trigono_metrico
 
Dfato vestibular fasciculo 5
Dfato vestibular fasciculo  5Dfato vestibular fasciculo  5
Dfato vestibular fasciculo 5
trigono_metrico
 
Ap trigonometria numeros complexo
Ap trigonometria numeros complexoAp trigonometria numeros complexo
Ap trigonometria numeros complexo
trigono_metrico
 
Apostila 3 calculo i integrais
Apostila 3 calculo i integraisApostila 3 calculo i integrais
Apostila 3 calculo i integrais
trigono_metrico
 
Dfato vestibular fasciculo 2
Dfato vestibular fasciculo  2Dfato vestibular fasciculo  2
Dfato vestibular fasciculo 2
trigono_metrico
 
Dfato vestibular fasciculo 4
Dfato vestibular fasciculo  4Dfato vestibular fasciculo  4
Dfato vestibular fasciculo 4
trigono_metrico
 
Apostila 2 calculo i derivadas
Apostila 2 calculo i derivadasApostila 2 calculo i derivadas
Apostila 2 calculo i derivadas
trigono_metrico
 
Mat exercicios resolvidos 011
Mat exercicios resolvidos  011Mat exercicios resolvidos  011
Mat exercicios resolvidos 011
trigono_metrico
 
Mat exercicios resolvidos 010
Mat exercicios resolvidos  010Mat exercicios resolvidos  010
Mat exercicios resolvidos 010
trigono_metrico
 

More from trigono_metrico (20)

Pro cefet fasciculo 03 resolução comentada
Pro cefet fasciculo 03 resolução comentadaPro cefet fasciculo 03 resolução comentada
Pro cefet fasciculo 03 resolução comentada
 
Pro cefet fasciculo 04 resolução comentada
Pro cefet fasciculo 04 resolução comentadaPro cefet fasciculo 04 resolução comentada
Pro cefet fasciculo 04 resolução comentada
 
Ap geometria resolvidos
Ap geometria resolvidosAp geometria resolvidos
Ap geometria resolvidos
 
Ap matemática m2
Ap matemática m2Ap matemática m2
Ap matemática m2
 
Ap geometria plana resolvidos
Ap geometria plana resolvidosAp geometria plana resolvidos
Ap geometria plana resolvidos
 
Ap matemática m3
Ap matemática m3Ap matemática m3
Ap matemática m3
 
Dfato vestibular fasciculo 3
Dfato vestibular fasciculo  3Dfato vestibular fasciculo  3
Dfato vestibular fasciculo 3
 
Apostila 3 funções
Apostila 3 funçõesApostila 3 funções
Apostila 3 funções
 
Ap geometria analitica resolvidos
Ap geometria analitica resolvidosAp geometria analitica resolvidos
Ap geometria analitica resolvidos
 
Dfato vestibular fasciculo 5
Dfato vestibular fasciculo  5Dfato vestibular fasciculo  5
Dfato vestibular fasciculo 5
 
Apostila 1 calculo i
Apostila 1 calculo iApostila 1 calculo i
Apostila 1 calculo i
 
Ap trigonometria numeros complexo
Ap trigonometria numeros complexoAp trigonometria numeros complexo
Ap trigonometria numeros complexo
 
Apostila 3 calculo i integrais
Apostila 3 calculo i integraisApostila 3 calculo i integrais
Apostila 3 calculo i integrais
 
Dfato vestibular fasciculo 2
Dfato vestibular fasciculo  2Dfato vestibular fasciculo  2
Dfato vestibular fasciculo 2
 
Apostila trigonometria
Apostila trigonometriaApostila trigonometria
Apostila trigonometria
 
Dfato vestibular fasciculo 4
Dfato vestibular fasciculo  4Dfato vestibular fasciculo  4
Dfato vestibular fasciculo 4
 
Apostila 2 calculo i derivadas
Apostila 2 calculo i derivadasApostila 2 calculo i derivadas
Apostila 2 calculo i derivadas
 
Mat exercicios resolvidos 011
Mat exercicios resolvidos  011Mat exercicios resolvidos  011
Mat exercicios resolvidos 011
 
Apostila 1 ec
Apostila 1 ecApostila 1 ec
Apostila 1 ec
 
Mat exercicios resolvidos 010
Mat exercicios resolvidos  010Mat exercicios resolvidos  010
Mat exercicios resolvidos 010
 

Recently uploaded

Aula 03 - Filogenia14+4134684516498481.pptx
Aula 03 - Filogenia14+4134684516498481.pptxAula 03 - Filogenia14+4134684516498481.pptx
Aula 03 - Filogenia14+4134684516498481.pptx
andrenespoli3
 
matematica aula didatica prática e tecni
matematica aula didatica prática e tecnimatematica aula didatica prática e tecni
matematica aula didatica prática e tecni
CleidianeCarvalhoPer
 
Responde ou passa na HISTÓRIA - REVOLUÇÃO INDUSTRIAL - 8º ANO.pptx
Responde ou passa na HISTÓRIA - REVOLUÇÃO INDUSTRIAL - 8º ANO.pptxResponde ou passa na HISTÓRIA - REVOLUÇÃO INDUSTRIAL - 8º ANO.pptx
Responde ou passa na HISTÓRIA - REVOLUÇÃO INDUSTRIAL - 8º ANO.pptx
AntonioVieira539017
 
Revolução russa e mexicana. Slides explicativos e atividades
Revolução russa e mexicana. Slides explicativos e atividadesRevolução russa e mexicana. Slides explicativos e atividades
Revolução russa e mexicana. Slides explicativos e atividades
FabianeMartins35
 
SSE_BQ_Matematica_4A_SR.pdfffffffffffffffffffffffffffffffffff
SSE_BQ_Matematica_4A_SR.pdfffffffffffffffffffffffffffffffffffSSE_BQ_Matematica_4A_SR.pdfffffffffffffffffffffffffffffffffff
SSE_BQ_Matematica_4A_SR.pdfffffffffffffffffffffffffffffffffff
NarlaAquino
 
8 Aula de predicado verbal e nominal - Predicativo do sujeito
8 Aula de predicado verbal e nominal - Predicativo do sujeito8 Aula de predicado verbal e nominal - Predicativo do sujeito
8 Aula de predicado verbal e nominal - Predicativo do sujeito
tatianehilda
 

Recently uploaded (20)

PROJETO DE EXTENSÃO I - SERVIÇOS JURÍDICOS, CARTORÁRIOS E NOTARIAIS.pdf
PROJETO DE EXTENSÃO I - SERVIÇOS JURÍDICOS, CARTORÁRIOS E NOTARIAIS.pdfPROJETO DE EXTENSÃO I - SERVIÇOS JURÍDICOS, CARTORÁRIOS E NOTARIAIS.pdf
PROJETO DE EXTENSÃO I - SERVIÇOS JURÍDICOS, CARTORÁRIOS E NOTARIAIS.pdf
 
Aula 03 - Filogenia14+4134684516498481.pptx
Aula 03 - Filogenia14+4134684516498481.pptxAula 03 - Filogenia14+4134684516498481.pptx
Aula 03 - Filogenia14+4134684516498481.pptx
 
PROJETO DE EXTENSÃO - EDUCAÇÃO FÍSICA BACHARELADO.pdf
PROJETO DE EXTENSÃO - EDUCAÇÃO FÍSICA BACHARELADO.pdfPROJETO DE EXTENSÃO - EDUCAÇÃO FÍSICA BACHARELADO.pdf
PROJETO DE EXTENSÃO - EDUCAÇÃO FÍSICA BACHARELADO.pdf
 
PRÁTICAS PEDAGÓGICAS GESTÃO DA APRENDIZAGEM
PRÁTICAS PEDAGÓGICAS GESTÃO DA APRENDIZAGEMPRÁTICAS PEDAGÓGICAS GESTÃO DA APRENDIZAGEM
PRÁTICAS PEDAGÓGICAS GESTÃO DA APRENDIZAGEM
 
matematica aula didatica prática e tecni
matematica aula didatica prática e tecnimatematica aula didatica prática e tecni
matematica aula didatica prática e tecni
 
Responde ou passa na HISTÓRIA - REVOLUÇÃO INDUSTRIAL - 8º ANO.pptx
Responde ou passa na HISTÓRIA - REVOLUÇÃO INDUSTRIAL - 8º ANO.pptxResponde ou passa na HISTÓRIA - REVOLUÇÃO INDUSTRIAL - 8º ANO.pptx
Responde ou passa na HISTÓRIA - REVOLUÇÃO INDUSTRIAL - 8º ANO.pptx
 
migração e trabalho 2º ano.pptx fenomenos
migração e trabalho 2º ano.pptx fenomenosmigração e trabalho 2º ano.pptx fenomenos
migração e trabalho 2º ano.pptx fenomenos
 
Currículo - Ícaro Kleisson - Tutor acadêmico.pdf
Currículo - Ícaro Kleisson - Tutor acadêmico.pdfCurrículo - Ícaro Kleisson - Tutor acadêmico.pdf
Currículo - Ícaro Kleisson - Tutor acadêmico.pdf
 
O PLANETA TERRA E SEU SATÉLITE NATURAL - LUA
O PLANETA TERRA E SEU SATÉLITE NATURAL - LUAO PLANETA TERRA E SEU SATÉLITE NATURAL - LUA
O PLANETA TERRA E SEU SATÉLITE NATURAL - LUA
 
Revolução russa e mexicana. Slides explicativos e atividades
Revolução russa e mexicana. Slides explicativos e atividadesRevolução russa e mexicana. Slides explicativos e atividades
Revolução russa e mexicana. Slides explicativos e atividades
 
Projeto de Extensão - ENGENHARIA DE SOFTWARE - BACHARELADO.pdf
Projeto de Extensão - ENGENHARIA DE SOFTWARE - BACHARELADO.pdfProjeto de Extensão - ENGENHARIA DE SOFTWARE - BACHARELADO.pdf
Projeto de Extensão - ENGENHARIA DE SOFTWARE - BACHARELADO.pdf
 
6ano variação linguística ensino fundamental.pptx
6ano variação linguística ensino fundamental.pptx6ano variação linguística ensino fundamental.pptx
6ano variação linguística ensino fundamental.pptx
 
SSE_BQ_Matematica_4A_SR.pdfffffffffffffffffffffffffffffffffff
SSE_BQ_Matematica_4A_SR.pdfffffffffffffffffffffffffffffffffffSSE_BQ_Matematica_4A_SR.pdfffffffffffffffffffffffffffffffffff
SSE_BQ_Matematica_4A_SR.pdfffffffffffffffffffffffffffffffffff
 
Texto dramático com Estrutura e exemplos.ppt
Texto dramático com Estrutura e exemplos.pptTexto dramático com Estrutura e exemplos.ppt
Texto dramático com Estrutura e exemplos.ppt
 
aula de bioquímica bioquímica dos carboidratos.ppt
aula de bioquímica bioquímica dos carboidratos.pptaula de bioquímica bioquímica dos carboidratos.ppt
aula de bioquímica bioquímica dos carboidratos.ppt
 
8 Aula de predicado verbal e nominal - Predicativo do sujeito
8 Aula de predicado verbal e nominal - Predicativo do sujeito8 Aula de predicado verbal e nominal - Predicativo do sujeito
8 Aula de predicado verbal e nominal - Predicativo do sujeito
 
Slides Lição 6, Betel, Ordenança para uma vida de obediência e submissão.pptx
Slides Lição 6, Betel, Ordenança para uma vida de obediência e submissão.pptxSlides Lição 6, Betel, Ordenança para uma vida de obediência e submissão.pptx
Slides Lição 6, Betel, Ordenança para uma vida de obediência e submissão.pptx
 
P P P 2024 - *CIEJA Santana / Tucuruvi*
P P P 2024  - *CIEJA Santana / Tucuruvi*P P P 2024  - *CIEJA Santana / Tucuruvi*
P P P 2024 - *CIEJA Santana / Tucuruvi*
 
About Vila Galé- Cadeia Empresarial de Hotéis
About Vila Galé- Cadeia Empresarial de HotéisAbout Vila Galé- Cadeia Empresarial de Hotéis
About Vila Galé- Cadeia Empresarial de Hotéis
 
Camadas da terra -Litosfera conteúdo 6º ano
Camadas da terra -Litosfera  conteúdo 6º anoCamadas da terra -Litosfera  conteúdo 6º ano
Camadas da terra -Litosfera conteúdo 6º ano
 

Mat em combinatoria sol vol2 cap4

  • 1. Solu¸˜es dos Exerc´ co ıcios da Se¸˜o 4.1 ca 1) A primeira pergunta pode ser respondida de 5 modos; a segunda, de 5 modos, etc. A resposta ´ 5 × 5 × · · · × 5 = 510 = 9.765.625. e 2) Para formar um subconjunto, deve-se decidir, para cada elemento do conjunto, se ele pertencer´ ou n˜o ao subconjunto. H´ 2 modos de a a a decidir o que fazer com o primeiro elemento do conjunto, 2 modos com o segundo, etc. A resposta ´ 2 × 2 × · · · × 2 = 2n . e Outra solu¸˜o: ca Quando se acrescenta um elemento a um conjunto, os subconjuntos do novo conjunto s˜o os subconjuntos do conjunto original e estes unidos a ao novo elemento. Ou seja, o n´ mero de subconjuntos dobra. Ent˜o, u a se An ´ o n´ mero de subconjuntos de um conjunto com n elementos, e u (An ) ´ uma progress˜o geom´trica de raz˜o 2. Logo, An = A0 .2n = 2n e a e a pois o conjunto vazio possui um unico subconjunto. ´ 3) A primeira pessoa tem 5 escolhas; a segunda, 4; a terceira, 3. A resposta ´ 5 × 4 × 3 = 60. e 4) Os bancos em que os homens se sentam podem ser escolhidos de 5×4× 3×2×1 = 120 modos, o mesmo ocorrendo com os bancos das mulheres. Em cada banco, os casais podem se sentar de 2 modos diferentes. A resposta ´ 1202 × 25 = 460.800. e 5) As 64 casas do tabuleiro dividem-se, naturalmente, em trˆs grupos: e i) as 4 casas dos v´rtices; e ii) as 24 casas da borda do tabuleiro, mas que n˜o s˜o v´rtices; a a e iii) as restantes 36 casas, que s˜o interiores ao tabuleiro. a Vamos separar a nossa contagem conforme o tipo de casa ocupada pelo rei negro: i) h´ 4 possibilidades para o rei negro e 60 para o rei branco; a ii) h´ 24 possibilidades para o rei negro e 58 para o rei branco; a
  • 2. iii) h´ 36 possibilidades para o rei negro e 55 para o rei branco. a A resposta ´ 4 × 60 + 24 × 58 + 36 × 55 = 3612. e Se os reis s˜o iguais, a resposta passa a ser a metade da resposta an- a terior, pois, trocando a posi¸˜o dos reis, agora obt´m-se a mesma con- ca e figura¸˜o. ca 6) Haver´ uma torre em cada linha e em cada coluna. A posi¸˜o da torre a ca da primeira linha pode ser escolhida de 8 modos; a da segunda linha, de 7, etc. A resposta ´ 8 × 7 × 6 × 5 × 4 × 3 × 2 × 1 = 40.320. e Se as torres fossem diferentes, para cada uma das escolhas de posi¸˜o, ca ter´ ıamos que escolher uma das torres. A resposta seria, portanto, 8 × 8 × 7 × 7 × 6 × 6 × 5 × 5 × 4 × ×3 × ×3 × 2 × 2 × 1 × 1 = (8!)2 = 1.625.702.400. 7) Se a primeira carta ´ o rei de copas, a segunda pode ser escolhida de 48 e modos (pode ser qualquer carta, exceto os 4 reis). Se a primeira carta ´e de copas mas n˜o ´ o rei, ela pode ser escolhida de 12 modos. Neste caso, a e a segunda carta pode ser escolhida de 47 modos (n˜o pode ser a primeira a escolhida, nem nenhum dos 4 reis). A resposta ´ 48 + 12 × 47 = 612. e 8) a) Para construir uma fun¸˜o, devemos, para cada elemento de A, es- ca colher sua imagem em B. H´ 7 modos de escolher a imagem do primeiro a elemento de A, 7 modos de escolher a imagem do segundo elemento, etc. A resposta ´ 7 × 7 × 7 × 7 = 2.401. e b) Para a fun¸˜o ser injetora, elementos diferentes devem ter imagens ca diferentes. H´ 7 modos de escolher a imagem do primeiro elemento de a A, 6 modos de escolher a imagem do segundo elemento, etc. A resposta ´ 7 × 6 × 5 × 4 = 840. e 9) a) Como 720 = 24 × 32 × 51 , 720 possui 5 × 3 × 2 = 30 divisores. Aos pares, estes divisores formam produtos iguais a 720. Logo, h´ 15 modos a de escrever 720 como um produto de divisores. b) Como 144 = 24 × 32 , 144 possui 3 × 3 = 9 divisores. Com eles, podem ser formados 4 pares de divisores cujo produto ´ 144 e, al´m e e 2
  • 3. disso, pode ser formado o produto 12 × 12. Assim, h´ 5 modos de a escrever 144 como um produto de divisores. 10) Um arm´rio ficar´ aberto se ele for mexido um n´mero ´ a a u ımpar de vezes. Por outro lado, o arm´rio de ordem k ´ mexido pelas pessoas cujos a e n´ meros s˜o divisores de k. Logo, estar˜o abertos os arm´rios cujos u a a a n´ meros possuem um n´ mero ´ u u ımpar de divisores. Isto ocorre com os n´ meros cujos expoentes s˜o todos pares na decomposi¸˜o em fatores u a ca primos, ou seja, s˜o quadrados perfeitos. Assim, permanecer˜o aber- a a tos os arm´rios cujos n´ meros s˜o quadrados perfeitos, ou seja, os de a u a n´ meros 12 , 22 , . . . , 302 . u 11) Separemos o caso em que o primeiro e o terceiro quadrantes tˆm cores e iguais do caso em que eles tˆm cores diferentes. e No caso de cores iguais, h´ 5 modos de escolher a cor unica para o a ´ primeiro e o terceiro quadrantes, 4 modos de escolher a cor para o segundo quadrante e 4 modos de escolher a cor para o quarto quadrante. H´, portanto, 5 × 4 × 4 = 80 modos de colorir o mapa usando cores a iguais no primeiro e no terceiro quadrantes. No caso de cores diferentes, h´ 5 modos de escolher a cor para o primeiro a quadrante, 4 modos de escolher a cor para o terceiro quadrante, 3 modos de escolher a cor para o segundo quadrante e 3 modos de escolher a cor para o quarto quadrante. H´ 5 × 4 × 3 × 3 = 180 modos de colorir o a mapa usando cores iguais no primeiro e no terceiro quadrantes. No total, temos, portanto, 80 + 180 = 260 modos de colorir a figura. 12) a) H´ 265 = 11.881.376 palavras de 5 letras. Delas, devemos subtrair a as palavras que come¸am por A, 1 × 264 = 456.976, e aquelas nas quais c a letra A n˜o figura, 255 = 9.765.625. a A resposta ´ 11.881.376 − 456.976 − 9.765.625 = 1.658.775. e b) O n´ mero total de palavras de 5 letras distintas ´ 26 × 25 × 24 × u e 23 × 22 = 7.893.600. Delas devemos subtrair as palavras que come¸am c por A, 1 × 25 × 24 × 23 × 22 = 303.600 e aquelas nas quais a letra A n˜o figura, 25 × 24 × 23 × 22 × 21 = 6.375.600. a A resposta ´ 7.893.600 − 303.600 − 6.375.600 = 1.214.400. e Outra solu¸˜o: ca 3
  • 4. H´ 4 posi¸˜es para colocar a letra A; depois disso, as quatro casas a co vazias podem ser preenchidas de 25, 24, 23 e 22 modos. A resposta ´ 4 × 25 × 24 × 23 × 22 = 1.214.400. e 13) Cada letra pode ser escolhida de 26 modos, enquanto cada algarismo pode ser escolhido de 10 modos. Logo, o n´ mero total de placas ´ u e 263 × 104 = 175.760.000. 14) O n´ mero de modos de acomodar os passageiros que pretendem sentar u de frente ´ 5 × 4 × 3 × 2 = 120; o n´ mero de modos de acomodar os e u passageiros que pretendem sentar de costas ´ 5 × 4 × 3 = 60; o n´mero e u de modos de acomodar os demais passageiros ´ 3 × 2 × 1 = 6. e A resposta ´ 120 × 60 × 6 = 43.200. e 15) Vamos primeiramente determinar quantos zeros s˜o escritos na casa a das unidades, depois na das dezenas, etc. H´ 222 n´ meros que tˆm 0 como algarismo das unidades, pois antes do a u e zero podem ser colocados os inteiros de 1 (inclusive) a 222 (inclusive). H´ 22 × 10 = 220 n´meros que tˆm 0 como algarismo das dezenas, a u e pois antes do zero podem ser colocados os inteiros de 1 (inclusive) a 22 (inclusive) e depois do zero, os inteiros de 0 (inclusive) a 9 (inclusive). H´ 2 × 100 = 200 n´meros que tˆm 0 como algarismo das centenas, a u e pois antes do zero podem ser colocados os inteiros de 1 (inclusive) a 2 (inclusive) e depois do zero, os inteiros de 0 (inclusive) a 99 (inclusive). A resposta ´ 222 + 220 + 200 = 642. e ´ 16) E mais simples contar, primeiramente, os n´ meros onde o algarismo u 5 n˜o aparece. O primeiro d´ a ıgito pode ser escolhido de 8 modos (n˜o a pode ser igual a 0 nem igual a 5) e cada um dos demais trˆs d´ e ıgitos pode ser selecionado de 9 modos (deve ser diferente de 5). Logo, h´ a 8×93 = 5.832 n´ meros de 4 algarismos em que n˜o aparece o algarismo u a 5. A quantidade de n´ meros de 4 d´ u ıgitos, com ou sem o d´ıgito 5, ´ 9×103 = e 9.000 (pois h´ 9 modos de selecionar o primeiro d´ a ıgito, que deve ser diferente de 0, e 10 modos de selecionar cada um dos demais 4 d´ ıgitos). Logo, h´ 9.000 − 5.832 = 3.168 n´ meros de 4 algarismos em que o 5 a u n˜o aparece. a 4
  • 5. 17) Devemos decidir quantos exemplares de cada revista devem ser postos na cole¸˜o. H´ 6 possibilidades para a ”Veja” (0, 1, 2, 3, 4, ou 5 ca a ´ exemplares), 7 para a ”Manchete” e 5 para a ”Isto E”. O n´ mero de u cole¸˜es ´ 6 × 7 × 5 = 210, e o n´mero de cole¸˜es n˜o-vazias ´ 209. co e u co a e 18) Em cada dia, duas das mat´rias s˜o ensinadas e uma folga. H´ 3 e a a possibilidades para escolher a mat´ria que folga na segunda, 2 para e escolher a que folga na quarta e 1 para escolher a que folga na sexta. Portanto, h´ 6 modos para escolher as mat´rias de cada dia. Para a e escolher os hor´rios, h´ 2 possibilidades em cada dia. Logo, o n´mero a a u total de hor´rios ´ 6 × 8 = 48. a e 19) Foi feita uma distin¸˜o artificial ao se considerar cada casal ordenado ca de dois modos diferentes: come¸ando pela mulher ou pelo homem. Por c esta raz˜o, o resultado encontrado foi igual ao dobro do correto. a 20) H´ trˆs tipos de cart˜es: os que virados de cabe¸a para baixo n˜o a e o c a representam n´ meros, como, por exemplo, 41.809; os que virados de u cabe¸a para baixo representam o mesmo n´mero, como, por exem- c u plo, 86.198; os que virados de cabe¸a para baixo representam n´meros c u diferentes, como, por exemplo, 66.810. Os cart˜es do ultimo tipo s˜o o ´ a os que permitem economia porque um mesmo cart˜o serve para rep- a resentar dois n´ meros. H´ 5 × 5 × 5 × 5 × 5 = 3.125 cart˜es que u a o virados de cabe¸a para baixo representam n´meros, iguais ou difer- c u entes, pois tais cart˜es devem ter como algarismos apenas 0, 1, 8, 6 o ou 9. Destes, 5 × 5 × 3 = 75 s˜o do segundo tipo, pois um tal cart˜o a a deve ter as casas das extremidades preenchidas por 00, 11, 88, 69 ou 96, a segunda e a quarta casas preenchidas por 00, 11, 88, 69 ou 96, e a casa central preenchida por 0, 1 ou 8. Portanto, os cart˜es do terceiro o tipo s˜o em n´ mero de 3.125 − 75 = 3.050. Podem ser economiza- a u dos 3.050/2 = 1.525 cart˜es. O n´ mero m´ o u ınimo de cart˜es de que se o necessita ´ 100.000 − 1.525 = 98.475. e 21) A decomposi¸˜o de 360 em fatores primos ´ 720 = 23 .32 .5. Os divisores ca e inteiros e positivos de 720 s˜o os n´ meros da forma 2α .3β .5γ , com α ∈ a u {0, 1, 2, 3}, β ∈ {0, 1, 2} e γ ∈ {0, 1}. A soma dos divisores ´ S = e α β γ Σ2 .3 .5 , com α ∈ {0, 1, 2, 3}, β ∈ {0, 1, 2} e γ ∈ {0, 1}. Para calcular essa soma, dividimos as parcelas em dois grupos, conforme seja γ = 0 ou γ = 1. S = Σ (2α .3β .50 ) + Σ (2α .3β .51 ) = 6 Σ (2α .3β ) porque a segunda 5
  • 6. soma ´ igual ao qu´ e ıntuplo da primeira. Agora, dividimos as parcelas em grupos, conforme seja β = 0, β = 1 ou β = 2. S = 6 [Σ (2α .30 ) + Σ (2α .31 ) + Σ (2α .32 )] = 6[Σ 2α + 3Σ 2α + 9Σ 2α ] = 6[13Σ 2α ] = 78Σ2α = 78[20 + 21 + 22 + 23 ] = 78 × 15 = 1.170. Solu¸˜es dos Exerc´ co ıcios da Se¸˜o 4.2 ca 1) a) O n´ mero total de anagramas ´ 8! = 40.320. u e b) H´ 4 modos de escolher a vogal que ser´ a primeira letra do anagrama a a e 3 modos de selecionar a vogal que ser´ a ultima letra do anagrama. a ´ Depois disso, h´ 6! modos de arrumar as demais letras entre a primeira a e a ultima. ´ A resposta ´ 4 × 3 × 6! = 4 × 3 × 720 = 8.640. e c) As vogais e consoantes podem aparecer na ordem CV CV CV CV ou na ordem VC VC VC VC. No primeiro caso, devemos colocar as 4 vogais nos 4 lugares de ordem par (4! modos) e as 4 consoantes nos 4 lugares de ordem ´ımpar (4! modos). H´ 4!×4! = 24 ×24 = 576 anagramas do primeiro tipo. Analogamente, a h´ 576 anagramas do segundo tipo. a A resposta ´ 576 + 576 = 1.152. e d) Tudo se passa como se CAP fosse uma unica letra. Devemos, por- ´ tanto, arrumar em fila 6 objetos: CAP,I,T,U,L,O. A resposta ´ 6! = 720. e e) Primeiramente, devemos escolher a ordem em que as letras C, A, P aparecer˜o. H´ 3! modos. Depois, devemos arrumar em fila 6 objetos: a a o bloco das letras C, A, P e as 5 letras I, T, U, L, O. H´ 6! modos. a A resposta ´ 3! × 6! = 6 × 720 = 4320. e f) Basta arrumar em fila, depois do PA, as restantes 6 letras. A resposta ´ 6! = 720. e g) H´ 7! anagramas com a letra P em primeiro lugar e h´ 7! anagramas a a com a letra A em segundo lugar. H´ tamb´m 6! anagramas com P a e em primeiro lugar e A em segundo lugar. Ao somarmos 7! com 7!, 6
  • 7. encontramos o n´ mero de anagramas com P em primeiro lugar ou A u em segundo lugar, mas contamos duas vezes os anagramas que tˆm P e em primeiro lugar e A em segundo lugar. A resposta ´ 7! + 7! − 6! = e 5040 + 5040 − 720 = 9.360. h) H´ 7! anagramas com a letra P em primeiro lugar, 7! anagramas a com a letra A em segundo e 7! anagramas com a letra C em terceiro. H´ tamb´m 6! anagramas com P em primeiro lugar e A em segundo a e lugar, 6! anagramas com P em primeiro e C em terceiro e 6! anagramas com A em segundo e C em terceiro. Finalmente, h´ 5! anagramas com a P em primeiro lugar, A em segundo e C em terceiro. Ao somarmos 7! com 7! com 7!, encontramos o n´mero de anagramas u que tˆm P em primeiro lugar ou A em segundo ou C em terceiro, mas e contamos alguns anagramas v´rias vezes. a Contamos duas vezes os anagramas que tˆm P em primeiro lugar e A em e segundo; o mesmo se deu com os que tˆm P em primeiro e C em terceiro e e com os que tˆm A em segundo e C em terceiro. Descontando essas e contagens indevidas, chegamos a 7! + 7! + 7! − 6! − 6! − 6! = 3.7! − 3.6!. Entretanto, anagramas com P em primeiro lugar e A em segundo e C em terceiro foram, inicialmente, contados trˆs vezes e, posteriormente, e descontados trˆs vezes, o que significa que n˜o est˜o sendo contados. e a a Incluindo-os na contagem, obtemos a resposta correta, que ´ 3.(7!) − e 3.(6!) + 5! = 3.(5.040) − 3.(720) + 120 = 13.080. i) Como h´ 6 ordens poss´ a ıveis para as letras C, A e P, os anagramas pedidos s˜o exatamente 1/6 do total, ou seja, 8!/6 = 6.720. a Outra solu¸˜o: ca Basta escolher as 3 posi¸˜es a serem ocupadas pelas Letras P, A, C, o co 3 que pode ser feito de C8 = 56 modos e distribuir as 5 letras restantes nas demais posi¸˜es, o que pode ser feito de 5! = 120 modos. O total co de anagramas ´ 56 × 120 = 6.720. e 2) O valor de f (a1 ) pode ser escolhido de n modos; o valor de f (a2 ), de n − 1 modos; . . . ; o de f (an ), de 1 modo. A resposta ´ n(n − 1) . . . 1 = n! e 3) O n´ mero total de modos de sentar 8 pessoas em 8 cadeiras ´ o n´ mero u e u de modos de arrumar 8 pessoas em fila, 8!. O n´mero de modos de u 7
  • 8. arrumar 8 pessoas em fila de modo que duas dessas pessoas, Vera e Paulo, fiquem juntas ´ 2.7!, pois, para formar uma tal fila, devemos e inicialmente decidir em que ordem se colocar˜o Vera e Paulo e, em a seguida, formar uma fila de 7 objetos: o bloco formado por Vera e Paulo; as demais 6 pessoas. A resposta ´ 8! - 2.7! = 40.320 - 10.080 = 30.240. e 4) Como visto no problema anterior, o n´mero de filas nas quais duas u pessoas (neste caso Helena e Pedro) ficam juntas ´ 2.7! = 10.080. O e n´ mero de filas onde Helena e Pedro e tamb´m Vera e Paulo ficam u e juntos ´ obtido de modo an´logo: agora s˜o dois blocos de duas pes- e a a soas, cada um podendo ser arrumado de dois modos distintos e mais 4 pessoas. Portanto, o n´mero de tais filas ´ 2.2.6! = 2.880. Logo, o u e n´ mero de filas em que Helena e Pedro ficam juntos, mas Vera e Paulo u n˜o, ´ 10.080 − 2.880 = 7.200. a e 5) O elemento da permuta¸˜o que ocupa o 10o lugar deve ser maior que 7. ca Pode ser escolhido de 3 modos. O elemento da 9a posi¸˜o deve ser maior ca que 6; haveria 4 possibilidades, mas uma delas j´ foi usada na escolha a do elemento que ocupa a 10a posi¸˜o. Pode ser escolhido de 3 modos. ca Prosseguindo com esse racioc´ ınio, vemos que a cada nova casa abranda- se a restri¸˜o, criando uma possibilidade a mais, mas ao mesmo tempo ca diminui-se uma possibilidade, pois uma delas foi usada na etapa. Ou seja, h´ 3 possibilidades para cada casa at´ a 3a casa. O elemento da a e 3a posi¸˜o deve ser maior que 3 − 3 = 0; haveria 10 possibilidades, ca mas 7 delas j´ foram usadas nas etapas anteriores. Pode ser escolhido a de 10 − 7 = 3 modos. O elemento da 2a posi¸˜o deve ser maior que ca 2 − 3 = −1; haveria 10 possibilidades, mas 8 delas j´ foram usadas nas a etapas anteriores. Pode ser escolhido de 10−8 = 2 modos. Finalmente, o elemento de posi¸˜o 1 deve ser maior que 1 − 3 = −2; haveria 10 ca possibilidades, mas 9 delas j´ foram usadas nas etapas anteriores. Pode a ser escolhido de 10 − 9 = 1 modo. A resposta ´ 38 .2.1 = 13.122. e a 5 5 6) H´ C15 modos de formar o Esporte; depois disso, C10 modos de formar o Tupi; finalmente, 1 unico modo de formar o Minas. ´ e 5 5 A resposta ´ C15 × C10 × 1 = 756.756. 7) O n´ mero de possibilidades ´ igual ao n´mero obtido no problema u e u 8
  • 9. anterior dividido por 3! = 6, j´ que permutando os nomes dos times a a subdivis˜o continua a mesma. A resposta ´ 756.756/6 = 126.126. a e 8) Escolha, sucessivamente, 3 pessoas para formar os 4 grupos de 3; isto 3 3 3 3 pode ser feito, sucessivamente, de C20 , C17 , C14 e C11 modos. A seguir, com as 8 pessoas restantes forme os 2 grupos restantes, o que pode ser 4 4 feito de C8 e C4 modos, respectivamente. Fazendo isso, contamos cada divis˜o 4!.2! vezes, porque, quando formamos os mesmos grupos de 3 a e os mesmos grupos 4 em outra ordem, contamos como se fosse outra divis˜o em grupos. a C 3 .C 3 .C 3 C 3 .C 4 .C 4 20! A resposta ´ 20 17 14 11 8 4 = e = 4!.3! (3!)4 (4!)2 4!2! = 67.897.830.000. Outra solu¸˜o: ca Forme uma fila com as 20 pessoas. Isso automaticamente as divide em 4 grupos de 3 e 2 grupo de 4: as 3 primeiras formam um grupo, as 3 seguintes formam outro, etc.. H´ 20! modos de formar a fila. a Entretanto, uma mesma divis˜o em grupos corresponde a v´rias filas a a diferentes, o que faz com que, no resultado 20!, cada divis˜o tenha a sido contada v´rias vezes. Devemos corrigir nossa contagem dividindo a o resultado pelo n´mero de vezes que cada divis˜o foi contada. Tro- u a cando a ordem dos elementos em cada grupo, o que pode ser feito de 3!.3!.3!.3!.4!.4! modos, ou a ordem dos grupos, o que pode ser feito de 4!.2! modos, a divis˜o em grupos n˜o se altera, mas a fila sim. Cada a a 20! divis˜o foi, assim, contada (6!)3 .2!.3! vezes e a resposta ´ a e 4 (4!)2 4!2! . (3!) 9) Os advers´rios em cada jogo podem ser escolhidos, sucessivamente, de a 2 2 2 2 2 2 C12 , C10 , C8 , C6 , C4 e C2 modos. No entanto, assim contamos cada poss´ rodada 6! vezes, j´ que contamos diferentes ordens dos jogos ıvel a C 2 .C 2 .C 2 .C 2 .C 2 .C 2 como se fossem rodadas diferentes. A resposta ´ 12 10 8 6 4 2 = e 6! 12! = 10.395 26 .6! Outra solu¸˜o: ca Colocando os 12 times em fila automaticamente formamos os 6 jogos da rodada. No entanto, a mesma rodada ´ contada v´rias vezes; os e a 9
  • 10. advers´rios em cada jogo podem ser ordenados de 2 modos, enquanto a os jogos podem ser ordenados de 6! modos. A resposta ´, portanto, e 12! . 26 .6! 10) a) Para determinar o lugar ocupado pelo n´mero 62.417, devemos con- u tar quantos n´meros est˜o antes dele. Antes dele est˜o 6.0023-380 u a a
  • 11. a 480 dezenas de milhar, ou seja, 4.800.000. A resposta ´ 480 + 4.800 e + 48.000 + 480.000 + 4.800.000 = 5.333.280. Outra solu¸˜o: ca H´ 5! = 120 parcelas na soma. Podemos agrup´-las em 60 pares, a a juntando a cada n´ mero o que dele se obt´m trocando o 1 com o 7, u e trocando o 2 com o 6, e conservando a posi¸˜o do 4. Em cada par, a ca soma vale 88.888. A resposta ´ 88.888 × 60 = 5.333.280. e 11) Devemos inicialmente escolher a ordem em que as mo¸as ficar˜o juntas, c a o que pode ser feito de m! maneiras. Em seguida, devemos arrumar em fila r + 1 objetos, os r rapazes e o bloco das mo¸as, o que pode ser c feito de (r + 1)! modos. A resposta ´ m!.(r + 1)!. e 12) a) A face a receber o n´ mero 1 pode ser escolhida de 6 modos, a do u n´ mero 2 de 5 modos, e assim por diante. O n´mero de possibilidades u u ´ 6.5.4.3.2.1 = 720. e b) Inicialmente, fazemos de conta que as faces tem cores diferentes. Contamos, pelo problema anterior, 720 dados. Como as faces s˜o in- a distingu´ıveis, o mesmo dado foi contado v´rias vezes. Por exemplo, a pense em um dado que tenha o 6 na face de baixo (face preta) e o 1 na face de cima (face branca). Ele ´, certamente, diferente de um dado e que tenha o 1 na face de baixo (face preta) e o 6 na face de cima (face branca). Mas sendo as faces indistingu´ ıveis, o dado que tem o 6 na face de baixo e o 1 na face de cima ´ igual ao dado que tem o 1 na face e de baixo e o 6 na face de cima; este ´, simplesmente, aquele de cabe¸a e c para baixo. Esse mesmo dado aparece outra vez com o 1 na face da frente e o 6 na face de tr´s, com o 1 na face da esquerda e o 6 na face da a direita, etc. Em suma, o mesmo dado foi contado tantas vezes quantas s˜o as posi¸˜es de coloc´-lo. a co a O n´ mero de posi¸˜es de colocar um cubo ´ 6×4 = 24, pois h´ 6 modos u co e a de escolher a face de baixo e 4 de escolher, nessa face, o lado que fica de frente. 720 A resposta ´e = 30. 24 11
  • 12. Outra solu¸˜o: ca Todo dado pode ser imaginado com a face 1 em baixo. Realmente, se o 1 n˜o estiver em baixo, ´ poss´ rodar o dado de modo que o 1 v´ a e ıvel a para baixo. Fixado o 1 em baixo, devemos escolher quem ocupar´ a a face oposta a face do 1. Isso pode ser feito de 5 modos. Digamos que ` tenha sido escolhido o 6. Com o 1 fixo em baixo e o 6 fixo em cima, devemos colocar os n´ meros 2, 3, 4 e 5 nas faces laterais. O 2 sempre u pode ser imaginado na face da frente. Com efeito, se o 2 n˜o estiver na a face da frente, uma conveniente rota¸˜o coloc´-lo-´ na face da frente, ca a a sem tirar o 1 da face de baixo nem o 6 da face de cima. Fixados o 2 na frente, o 1 em baixo e o 6 em cima, devemos escolher quem ocupar´ a a face oposta a face do 2. Isso pode ser feito de 3 modos. Digamos que ` tenha sido escolhido o 4. Agora, devemos colocar o 3 e o 5 nas faces da direita e da esquerda. Note que qualquer movimento com o dado ou retirar´ o 1 de baixo, ou o 6 de cima, ou o 2 da frente, ou o 4 de tr´s. a a Portanto, h´ 2 modos de preencher as faces direita e esquerda com os a n´ meros 3 e 5. u A resposta ´ 5 × 3 × 2 = 30. e c) Um dado com faces de cores diferentes pode, agora, ser numerado de apenas 6.4.2 = 48 modos, j´ que temos 6 faces a escolher para o a n´ mero 1 (isto determina a face do n´mero 6), 4 para o n´mero 2 (o u u u que determina a face do 5) e 2 para o n´mero 3 (que determina a do u 4). Mas como as faces s˜o iguais, cada dado ´ contado, como no item a e anterior, 24 vezes. Logo h´ apenas 48/24 = 2 dados distintos. a Outra Solu¸˜o: ca Como antes, podemos fixar o dado com o n´ mero 1 em baixo. Agora, u no entanto, isto tamb´m fixa o n´ mero 6 na face de cima. Agora, o e u n´ mero 2 pode ser fixado na face da frente (e, portanto, o n´ mero 5 na u u de tr´s). Assim, tudo que temos a escolher ´ se a face lateral da direita a e ´ o 3 ou o 4. Temos, portanto, apenas duas possibilidades. e 13) a) O n´ mero de posi¸˜es para um tetraedro regular ´ 4 × 3 = 12, pois u co e h´ 4 modos de escolher a face de apoio e 4 de escolher, nessa face, o a lado que fica de frente. 4! A resposta ´ e = 2. 12 12
  • 13. b) O n´ mero de posi¸˜es para um octaedro regular ´ 6 × 4 = 24, pois u co e h´ 6 modos de escolher o v´rtice de apoio e 4 de escolher, dentre as a e arestas que incidem nesse v´rtice, a que fica de frente. e 8! A resposta ´ e = 1.680. 24 c) O n´ mero de posi¸˜es para um dodecaedro regular ´ 12 × 5 = 60, u co e pois h´ 12 modos de escolher a face de apoio e 5 de escolher, nessa face, a o lado que fica de frente. 12! A resposta ´ e = 7.983.360. 60 d) O n´ mero de posi¸˜es para um icosaedro regular ´ 20 × 3 = 60, pois u co e h´ 20 modos de escolher a face de apoio e 3 de escolher, nessa face, o a lado que fica de frente. 20! A resposta ´ e = 40.548.366.802.944.000 ∼ 4.1016 . = 60 14) Temos 1! = 1, que ´ um quadrado perfeito, 1! + 2! = 1 + 2 = 3, que n˜o e a ´ quadrado perfeito, 1! + 2! + 3! = 1 + 2 + 6 = 9, que novamente ´ um e e quadrado perfeito, 1! + 2! + 3! + 4! = 1 + 2 + 6 + 24 = 33, que n˜o ´ a e quadrado perfeito. Todos os fatoriais seguinte, a partir de 5! terminam com zero, j´ que s˜o m´ ltiplos de 5 e 2. Logo, todas as somas da forma a a u n 1 k! para n ≥ 5 terminam com o algarismo 3 e n˜o s˜o, portanto, a a quadrados perfeitos. As unicas solu¸˜es s˜o n = 1 e n = 3. ´ co a 15) Em ESTRELADA as letras A e E aparecem 2 vezes cada e as letras S, T, R, L e D aparecem 1 vez cada uma, havendo, portanto, 9 letras na palavra. Para formar um anagrama, devemos escolher 2 das 9 posi¸˜es paraco 2 colocar as letras A, o que pode ser feito de C9 modos, 2 das 7 posi¸˜es co 2 restantes para colocar as letras E, o que pode ser feito de C7 modos, e arrumar as letras S, T, R, L e D nas 5 posi¸˜es restantes, o que pode co 2 2 ser feito de 5! modos. A resposta ´ C9 .C7 .5! = 90.720. e Outra solu¸˜o: ca 9! u e 2,2,1,1,1,1,1 = O n´ mero de anagramas ´ P9 = 90.720. 2!2!1!1!1!1!1! 16) Formar um subconjunto com p elementos significa escolher p dos n e p elementos. A resposta ´ Cn . 13
  • 14. 4 17) Basta escolher as provas do primeiro dia, o que pode ser feito de C8 = 70 modos. 18) O processo de contagem apresentado conta determinadas comiss˜es o mais de uma vez. Isto ocorre porque um homem que participe da comiss˜o pode ser inserido de dois modos diferentes: como um dos 3 a homens escolhidos inicialmente, ou como uma das duas pessoas escol- hidas posteriormente. O pior ´ que n˜o ´ poss´ e a e ıvel ”corrigir” a con- tagem dividindo pelo n´mero de vezes que cada comiss˜o ´ contada: u a e as comiss˜es com 3 homens s˜o contadas apenas uma vez, as que tˆm o a e 4 homens s˜
  • 15. 4(4 − 3) do poliedro, 12 dos quais s˜o arestas e 6 a = 12 dos quais s˜o a 2 diagonais de faces. A resposta ´ 28 − 12 − 12 = 4. e Outra solu¸˜o: ca Cada diagonal de um prisma n-agonal une um v´rtice da base “de e cima” a um v´rtice da base “de baixo”. O v´rtice da base “de cima” e e pode ser selecionado de n modos; depois disso, o da base ”de baixo” pode ser selecionado de n − 3 modos, pois um dos v´rtices da base ”de e baixo”, se selecionado, daria origem a uma aresta e os dois v´rtices a e ele adjacentes nesta base dariam origem a diagonais de faces laterais. O n´ mero de diagonais de um prisma n-agonal ´, portanto, n(n − 3). u e Como o cubo ´ um prisma quadrangular, e a resposta ´ 4(4 − 3) = 4. e e) O prisma hexagonal ´ um poliedro formado por 6 faces quadran- e gulares e 2 faces hexagonais e que tem 12 v´rtices e 18 arestas. H´ e a 2 C12 = 66 segmentos que unem dois v´rtices do poliedro, 18 dos quais e 4(4 − 3) 6(6 − 3) s˜o arestas e 6 a +2 = 30 dos quais s˜o diagonais de a 2 2 faces. A resposta ´ 66 − 18 − 30 = 18. e Outra solu¸˜o: ca O n´ mero de diagonais de um prisma n-agonal ´, como visto em d), u e n(n − 3). Portanto, o n´ mero de diagonais de um prisma hexagonal ´ u e 6(6 − 3) = 18. 20) Uma fun¸˜o estritamente crescente ´ necessariamente injetiva (se f (a) = ca e f (b), n˜o pode ser a < b, pois, neste caso, f (a) < f (b), o que ´ absurdo; a e do mesmo modo, n˜o pode ser a > b, pois, neste caso, f (a) > f (b), a o que ´ absurdo; logo, a = b). Logo, seu conjunto de valores ter´ e a exatamente m elementos. Para construir uma tal fun¸˜o, devemos, ini- ca m cialmente, selecionar o conjunto de valores, o que pode ser feito de Cn modos. Selecionado o conjunto de valores, a fun¸˜o est´ determinada porque ca a f (1) deve ser igual ao menor elemento do conjunto de valores, f (2) deve ser igual ao segundo menor elemento do conjunto de valores, etc. m A resposta ´, portanto, Cn . e 15
  • 16. 21) Vamos esquecer que a primeira casa do n´mero n˜o pode ser igual a u a zero. Isso far´ com que contemos a mais e, depois, descontaremos o a que foi contado indevidamente. a 3 H´ C7 modos de escolher as casas que ser˜o ocupadas pelo d´ a ıgito 4; a 2 depois disso, h´ C4 modos de selecionar as casas que ser˜o ocupadas a pelo d´ ıgito 8; finalmente, as duas casas restantes podem ser preenchidas de 8 × 8 modos (n˜o podemos usar nessas casas os d´ a ıgitos 4 e 8). 3 2 A “resposta” seria C7 × C4 × 8 × 8 = 35 × 6 × 64 = 13.440. Devemos subtrair os n´meros come¸ados por 0. Se o n´mero come¸a u c u c 3 por 0, h´ C6 modos de escolher as casas que ser˜o ocupadas pelo d´ a a ıgito a 2 4; depois disso, h´ C3 modos de selecionar as casas que ser˜o ocupadas a pelo d´ıgito 8; finalmente, a casa restante pode ser preenchida de 8 modos (n˜o podemos usar nessa casa os d´ a a 3 2 ıgitos 4 e 8). H´ C6 ×C3 ×8 = 20 × 3 × 8 = 480 n´meros come¸ados por 0. u c A resposta ´ 13.440 − 480 = 12.960. e Outra solu¸˜o: ca Vamos contar separadamente: i) n´ meros que come¸am em 4; ii) n´ meros que come¸am em 8; iii) u c u c n´ meros que n˜o come¸am nem em 4 nem em 8. u a c i) H´ 1 modo de preencher a primeira casa; depois disso, h´ C6 a a 2 modos de escolher as outras duas casas do n´ mero que tamb´m u e a 2 ser˜o preenchidas com o algarismo 4; depois disso, h´ C4 modos a de escolher as duas casas que ser˜o ocupadas pelo algarismo 8; a finalmente, as duas casas restantes podem ser preenchidas de 8×8 modos (n˜o podemos usar nessas casas os d´ a ıgitos 4 e 8). 2 2 H´ 1 × C6 × C4 × 8 × 8 = 1 × 15 × 6 × 64 = 5.760 n´ meros do a u tipo i). ii) H´ 1 modo de preencher a primeira casa; depois disso, h´ 6 modos a a de escolher a outra casa do n´ mero que tamb´m ser´ preenchida u e a a 3 com o algarismo 8; depois disso, h´ C5 modos de escolher as trˆs e casas que ser˜o ocupadas pelo algarismo 4; finalmente, as duas a casas restantes podem ser preenchidas de 8 × 8 modos (n˜o pode- a mos usar nessas casas os d´ ıgitos 4 e 8). 3 H´ 1 × 6 × C5 × 8 × 8 = 6 × 10 × 64 = 3840 n´meros do tipo ii). a u 16
  • 17. iii) H´ 7 modos de preencher a primeira casa (n˜o podemos usar nem a a 3 4, nem 8, nem 0); depois disso, h´ C6 modos de escolher as trˆs a e casas do n´ mero que ser˜o preenchidas com o algarismo 4; depois u a a 2 disso, h´ C3 modos de escolher as duas casas que ser˜o ocupadas a pelo algarismo 8; finalmente, a casa restante pode ser preenchida de 8 modos (n˜o podemos usar nessas casas os d´ a ıgitos 4 e 8). 3 2 H´ 7 × C6 × C3 × 8 = 7 × 20 × 3 × 8 = 3.360 n´ meros do tipo iii). a u A resposta ´ 5.760 + 3.840 + 3.360 = 12.960. e 22) a) Para formar o subconjunto devemos escolher os p−1 outros elementos do subconjunto dentre os n − 1 outros elementos do conjunto. e p−1 A resposta ´ Cn−1 . b) Para formar o subconjunto devemos escolher os p elementos do sub- conjunto dentre os n − 1 outros elementos do conjunto. e p A resposta ´ Cn−1 . Outra solu¸˜o: ca p−1 a p H´ Cn p-subconjuntos e o elemento a1 figura em Cn−1 deles. Logo, h´ a p p−1 Cn − Cn−1 subconjuntos nos quais o elemento a1 n˜o figura. a p−1 e p A resposta ´ Cn − Cn−1 . p p−1 Observa¸˜o: As duas solu¸˜es apresentadas mostram que Cn −Cn−1 . = ca co p Cn−1 . Essa ´ a famosa Rela¸˜o de Stifel. e ca c) Para formar o subconjunto devemos escolher os p−2 outros elementos do subconjunto dentre os n − 2 outros elementos do conjunto. e p−2 A resposta ´ Cn−2 . e p d) O total de p-subconjuntos ´ Cn . Para formar um subconjunto em que nem a1 nem a2 figurem devemos escolher os p elementos do subconjunto p dentre os n − 2 outros elementos do conjunto. H´, portanto, Cn−2 a subconjuntos nos quais nem a1 nem a2 figuram. Logo, o n´mero de u subconjuntos nos quais pelo menos um desses dois elementos figura ´ e p p Cn − Cn−2 . Outra solu¸˜o: ca a p−1 a p−1 H´ Cn−1 p-subconjuntos nos quais o elemento a1 figura e h´ Cn−1 p−2 subconjuntos nos quais o elemento a2 figura. H´, tamb´m, Cn−2 p- a e 17
  • 18. subconjuntos nos quais os elementos a1 e a2 figuram ambos. Ao so- p−1 p−1 p−1 marmos Cn−1 + Cn−1 = 2Cn−1 obtemos o n´ mero de subconjuntos nos u quais pelo menos um dos elementos a1 e a2 figura, mas contamos duas vezes aqueles em que a1 e a2 figuram ambos. p−1 p−2 A resposta ´, portanto, 2Cn−1 − Cn−2 . e Outra solu¸˜o: ca p−2 H´, como mostrado em c), Cn−2 p-subconjuntos em que os elementos a a1 e a2 figuram ambos. a p−1 H´ Cn−2 p-subconjuntos em que o elemento a1 figura e o elemento a2 n˜o figura, pois, para formar um tal subconjunto, basta escolher os a outros p − 1 elementos do subconjunto dentre os n − 2 elementos do conjunto que s˜o diferentes de a1 e de a2 . a p−1 H´, analogamente, Cn−2 p-subconjuntos em que o elemento a2 figura a e o elemento a1 n˜o figura. Portanto, o n´mero de p-subconjuntos em a u p−1 p−2 que figura pelo menos um desses dois elementos ´ 2Cn−2 + Cn−2 . e p−1 e) Como visto na solu¸˜o anterior, a resposta ´ 2Cn−2 . ca e Outra solu¸˜o: ca p−1 p−2 H´, como visto em d), 2Cn−1 − Cn−2 p-subconjuntos nos quais pelo a p−2 menos um dos elementos a1 e a2 figura. H´, como visto em c), Cn−2 a p-subconjuntos em que os elementos a1 e a2 figuram ambos. p−1 p−2 p−2 A resposta ´, portanto, 2Cn−1 − Cn−2 − Cn−2 = e p−1 p−2 = 2Cn−1 − 2Cn−2. Outra solu¸˜o: ca p−1 p−2 H´, como visto em d), 2Cn−2 + Cn−2 p-subconjuntos nos quais pelo a p−2 menos um dos elementos a1 e a2 figura. H´, como visto em c), Cn−2 a p-subconjuntos em que os elementos a1 e a2 figuram ambos. p−1 p−2 p−2 A resposta ´, portanto, 2Cn−2 + Cn−2 − Cn−2 = e p−1 = 2Cn−2 . e 5 23) a) Como h´ 32 cartas, a resposta ´ C32 = 201.376. a 18
  • 19. b) H´ 8 modos de escolher o grupo do par propriamente dito (por a 2 exemplo, valete), C4 modos de escolher os naipes das duas cartas do 3 par (por exemplo, copas e paus), C7 modos de escolher os grupos das outras trˆs cartas (por exemplo, 10, 8 e rei) e 4 × 4 × 4 = 43 modos de e escolher os naipes dessas trˆs cartas. e A resposta ´ 8 × C4 × C7 × 43 = 107.520. e 2 3 a 2 c) H´ C8 modos de escolher os grupos das cartas que formar˜o os dois a 2 2 pares, (C4 ) modos de escolher seus naipes, 6 modos de escolher o grupo da outra carta e 4 modos de escolher seu naipe. A resposta ´ C8 × (C4 )2 × 6 × 4 = 24.192. e 2 2 Observa¸˜o: Um erro muito comum ´ o exposto a seguir. ca e 2 H´ 8 modos de escolher o grupo do primeiro par, C4 modos de escolher a os naipes do primeiro par, 7 modos de escolher o grupo do segundo par, 2 C4 modos de escolher os naipes do segundo par, 6 modos de escolher o grupo da outra carta e 4 modos de escolher seu naipe. A resposta 2 2 ERRADA seria 8 × C4 × 7 × C4 × 6 × 4 = 48.384. A explica¸˜o do ca ERRO ´ simples: Ao fazermos a inexistente distin¸˜o entre primeiro e ca par e segundo par, contamos pares de valetes e reis como diferentes de pares de reis e valetes. A resposta ERRADA pode ser corrigida dividindo-a por 2. 3 d) H´ 8 modos de escolher o grupo da trinca, C4 modos de escolher os a 2 naipes das cartas da trinca, C7 modos de escolher os grupos das outras duas cartas e 4×4 = 42 modos de escolher os naipes dessas duas cartas. A resposta ´ 8 × C4 × C7 × 42 = 10.752. e 3 2 e) H´ 8 modos de escolher o grupo do ”four”, 1 modo de escolher os a naipes das quatro cartas do ”four”, 7 modos de escolher o grupo da outra carta e 4 modos de escolher o naipe dessa carta. A resposta ´ 8 × 1 × 7 × 4 = 224. e 3 f) H´ 8 modos de escolher o grupo da trinca, C4 modos de escolher os a 2 naipes das cartas da trinca, 7 modos de escolher o grupo do par e C4 modos de escolher os naipes das cartas do par. 3 2 A resposta ´ 8 × C4 × 7 × C4 = 1.344. e g) H´ apenas 4 tipos de seq¨ˆncias: 7, 8, 9, 10, valete; 8, 9, 10, valete, a ue dama; 9, 10, valete, dama, rei; 10, valete, dama, rei, as. Escolhido o ´ 19
  • 20. tipo da seq¨ˆncia, haveria 4 × 4 × 4 × 4 × 4 = 45 modos de escolher os ue naipes das cartas da seq¨ˆncia, mas 4 desses modos n˜o s˜o permitidos: ue a a todas de ouros, todas de paus, todas de copas e todas de espadas. A resposta ´ 4 × [45 − 4] = 4.080. e 5 h) Os grupos das cartas podem ser escolhidos de C8 − 4 modos e o naipe unico, de 4 modos. ´ e 5 A resposta ´ (C8 − 4) × 4 = 208. i) H´ 4 modos de escolher os grupos das cartas e 4 modos de escolher a o naipe unico. ´ A resposta ´ 4 × 4 = 16. e j) H´ 4 modos de escolher o naipe unico. A resposta ´ 4. a ´ e 24) a) Neste caso f ´ bijetiva e, se #A = #B = n, o n´ mero de fun¸˜es e u co f : A → B bijetivas ´ n!, como foi mostrado no exerc´ 4 da se¸˜o e ıcio ca 2.2. b) Neste caso dois elementos de A ter˜o uma mesma imagem em B e a a correspondˆncia entre os demais n − 1 elementos de A e os demais e n − 1 elementos de B ser´ bijetiva. a n+1 H´ a modos de escolher os dois elementos de A, n modos de 2 escolher a imagem deles em B e (n − 1)! modos de construir uma correspondˆncia bijetiva entre os elementos restantes. e n+1 n(n + 1)! A resposta ´e · n · (n − 1)! = . 2 2 c) Neste caso temos as alternativas: i) Trˆs elementos de A tˆm a mesma imagem em B e a correspondˆncia e e e entre os demais n − 1 elementos de A e os demais n − 1 elementos de B ´ bijetiva. e n+2 H´a modos de escolher os trˆs elementos de A, n modos e 3 de escolher a imagem deles em B e (n − 1)! modos de construir uma correspondˆncia bijetiva entre os elementos restantes. e n+2 n(n + 2)! H´a .n.(n − 1)! = fun¸˜es desse tipo. co 3 6 20
  • 21. ii) H´ dois pares de elementos de A com imagens idˆnticas em B e a a e correspondˆncia entre os demais n − 2 elementos de A e os demais e n − 2 elementos de B ´ bijetiva. e n H´a modos de escolher os dois elementos de 2 n+2 n B, × modos de escolher suas imagens inversas em A 2 2 e (n − 2)! modos estabelecer a correspondˆncia entre os elementos e restantes. n n+2 n n(n − 1)(n + 2)! H´ a × × × (n − 2)! = fun¸˜es desse co 2 2 2 8 tipo. n(n + 2)! n(n − 1)(n + 2)! n(3n + 1)(n + 2)! A resposta ´ e + = . 6 8 24 25) Chamemos de D o conjunto C − C1 . H´ quatro tipos de planos: a i) determinados por trˆs pontos de D; e ii) determinados por dois pontos de D e um de C1 ; iii) determinados por um ponto de D e dois de C1 ; iv) determinados por trˆs pontos de C1 . e e 3 2 2 A resposta ´ C12 + C12 .8 + 12.C8 + 1 = 1.085. Outra solu¸˜o: ca Para determinar um plano, devemos selecionar 3 dos 20 pontos, o que 3 pode ser feito de C20 = 1140 modos. Nessa contagem, o plano que 3 cont´m os 8 pontos de C1 foi contado C8 = 56 vezes. e A resposta ´ 1.140 − 56 + 1 = 1.085. e 26) Escolhida a ordem de cada casal, o que pode ser feito de 23 modos temos que arrumar em fila 4 espa¸os vazios e 3 casais, o que pode ser c 4 feito de C7 modos (escolha dos espa¸os vazios) vezes 3! (coloca¸˜o dos c ca 3 casais nos 3 lugares restantes). 21
  • 22. A resposta ´ 23 × C7 × 3! = 1.680. e 4 27) Primeiro, colocamos as vogais. Como a letra A aparece 3 vezes e as letras U, I e O aparecem 1 vez cada, o n´ mero de modos de dispˆ-las u o 6! 720 e 3,1,1,1 = ´ P6 = = 120. A seguir, colocamos as consoantes em 3! 6 trˆs dos 7 espa¸os antes, entre e depois das vogais. O lugar do P pode e c ser qualquer um destes 7 espa¸os, o do R qualquer dos 6 restantes e c o do G qualquer dos 5 restantes. O n´ mero total de possibilidades ´ u e 120.7.6.5 = 25.200. 28) Vamos formar uma fila com os n´ meros 1, 2, . . . n e assinalar com E os u p n´ meros escolhidos e com N os n − p n˜o escolhidos. A condi¸˜o u a ca para que n˜o sejam escolhidos n´meros consecutivos ´ que entre dois E a u e haja pelo menos um N. Come¸amos escrevendo os n − p E. A seguir, c devemos escolher, para colocar os E, p dentre os n − p + 1 espa¸os c p situados antes, entre e depois dos N. Isto pode ser feito de Cn−p+1 modos. 29) Chegam 4 cientistas A, B, C, D. Com as chaves que possuem, abrem alguns cadeados, mas n˜o todos. Existe pelo menos um cadeado que a eles n˜o conseguem abrir. Na situa¸˜o do n´ mero m´ a ca u ınimo de cadeados, existe exatamente um cadeado que eles n˜o conseguem abrir. Batize a tal cadeado de ABCD. Portanto, ABCD ´ o cadeado cuja chave n˜o e a est´ em poder de A, nem de B, nem de C e nem de D. Qualquer outro a cientista tem a chave desse cadeado, pois esse cientista e A, B, C e D formam um grupo de 5 cientistas e, portanto, nesse grupo algu´m e possui a chave. Como o algu´m n˜o ´ nem A, nem B, nem C e nem D, e a e deve ser o outro. Analogamente batize os demais cadeados. Verifique agora que a correspondˆncia entre cadeados e seus nomes ´ biun´ e e ıvoca, isto ´, cadeados diferentes tˆm nomes diferentes (isso porque estamos e e na situa¸˜o do n´ mero m´ ca u ınimo de cadeados) e cadeados de nomes difer- entes s˜o diferentes (se X est´ no nome de um cadeado e n˜o est´ no a a a a nome do outro, X tem a chave deste e n˜o tem a chave daquele). a a) O n´ mero m´ u ınimo de cadeados ´ igual ao n´mero de nomes de cadea- e u 4 dos, C1 1 = 330. b) Cada cientista X possui as chaves dos cadeados que n˜o possuem X a 4 no nome, C1 0 = 210. 22
  • 23. 30) Nenhum aluno pode comparecer a mais de trˆs jantares. Com efeito, se e A1 vai a um jantar com A2 e A3 , ele s´ pode ir a outro jantar com outros o dois estudantes, digamos A4 e A5 e s´ pode ir a um terceiro jantar em o companhia de outros dois, digamos A6 e A7 e n˜o ter´ companhia para a a ir a um quarto jantar. Como h´ 21 convites e s˜o 7 estudantes, cada a a estudante ter´ que comparecer a exatamente 3 jantares. a Se A1 comparece a trˆs jantares, podemos escolher os seus compan- e heiros dividindo os outros 6 estudantes em 3 grupos de 2, o que pode C 2 × C4 × 1 2 ser feito de 6 = 15 modos. 3! Ent˜o, os 3 jantares s˜o, digamos, A1 A2 A3 , A1 A4 A5 , A1 A6 A7 . a a A2 dever´ comparecer a mais dois jantares, nenhum deles em com- a panhia de A3 , e A3 tamb´m dever´ comparecer a mais dois jantares. e a Portanto, os 4 jantares que faltam s˜o: a A2 , A2 , A3 , A3 Como A4 deve comparecer a mais dois jantares (A4 n˜o pode compare- a cer a ambos em companhia de A2 nem a ambos em companhia de A3 ), esses quatro jantares s˜o:. a A2 A4 , A2 , A3 A4 , A3 ; A5 tem que comparecer ainda a dois jantares, nenhum deles em com- panhia de A4 . A2 A4 , A2 A5 , A3 A4 , A3 A5 . Agora h´ duas possibilidades: a A2 A4 A6 , A2 A5 A7 , A3 A4 A7 , A3 A5 A6 e A2 A4 A7 , A2 A5 A6 , A3 A4 A6 , A3 A5 A7 . H´ portanto 15 × 2 = 30 maneiras de escolher os grupos de convidados. a Para distribuir os 7 grupos nos 7 dias, h´ 7! alternativas. a A resposta ´ 7! × 30 = 151.200. e 31) Os dois primeiros lugares s´ podem ser ocupados por elementos de o {a1 , a2 , · · · a7 } e os dois ultimos por elementos de {a9 , a10 , a11 , a12 }. ´ e 2 2 A resposta ´ C7 × C4 = 126. 23
  • 24. a m 32) H´ Cm+h × 1 modos de escolher os lugares para os homens. Feito isso, s´ h´ 1 modo de formar a fila. o a e m m A resposta ´ Cm+h × 1 = Cm+h . 33) a) Cada professor fica caracterizado pelas duas bancas a que pertence. O n´ mero de professores ´ igual ao n´mero de modos de escolher duas u e u das oito bancas. e 2 A resposta ´ C8 = 28. b) O n´ mero de professores pertencentes a uma banca ´ igual ao n´mero u e u de modos de escolher a outra banca a que ele pertence. A resposta ´ 7. e 34) a) Imagine um quadro em que cada linha ´ a rela¸˜o dos atletas de e ca um time. O n´ mero de elementos do quadro ´ o n´ mero de times, u e u t, multiplicado pelo tamanho de cada time, k, e ´ tamb´m igual ao e e n´ mero de atletas, a, multiplicado pelo n´ mero de times de que cada u u atleta participa, x. tk Logo, ax = tk e x = . a b) No mesmo quadro, o n´mero de pares de atletas na mesma linha u ´ igual ao n´mero de linhas, t, multiplicado pelo n´ mero de pares de e u u 2 atletas em uma linha, Ck , e ´ tamb´m igual ao n´mero de pares de e e u 2 atletas, Ca , multiplicado pelo n´ mero de times em que cada par de u atletas fica junto, y. 2 2 2 tCk tk(k − 1) Logo, yCa = tCk e y = 2 = . Ca a(a − 1) 35) A resposta ´ o n´ mero de permuta¸˜es circulares de 4 elementos, ou e u co seja, 3! = 6. 36) H´ (P C)5 = 4! modos de formar uma roda com as meninas. Depois a disso, os 5 meninos devem ser postos nos 5 lugares entre as meninas, o que pode ser feito de 5! modos. A resposta ´ 4!×5! = 24×120 = 2.880. e ´ 37) E mais simples calcular o n´ mero total de rodas e excluir aquelas em u que Vera e Isadora ficam juntas. O n´ mero total de rodas ´ P C6 = u e 5! = 120. Para formar as rodas em que Vera e Isadora ficam juntas, a 24
  • 25. primeira decis˜o a tomar ´ a ordem em que Vera e Isadora se colocar˜o a e a na roda. H´ 2 possibilidades: Vera-Isadora e Isadora-Vera. Agora tudo a se passa como se Vera e Isadora fossem uma unica crian¸a. Assim, ´ c h´ 2(P C)5 = 2.4! = 48 rodas em que Vera e Isadora ficam juntas. A a resposta ´ 120 − 48 = 72 rodas. e 38) Chamando x de 1 + a, y de 1 + b e z de 1 + c, o problema se transforma em encontrar todas as solu¸˜es inteiras e n˜o- negativas de (a+1)+(b+ co a 4 4 1) + (c + 1) = 7, ou seja, de a + b + c = 4. A resposta ´ CR3 = C6 = 15. e 39) Cada solu¸˜o inteira e n˜o negativa da inequa¸˜o x + y + z ≤ 6 corre- ca a ca sponde a uma solu¸˜o inteira e n˜o negativa da equa¸˜o x+y+z+f = 6. ca a ca 6 6 Logo, h´ CR4 = C9 = 84 solu¸˜es. a co 40) Para formar uma caixa, devemos selecionar 20 dentre os 5 tipos, valendo repeti¸˜o na escolha. Ou seja, devemos formar solu¸˜es inteiras e n˜o ca co a negativas de x1 +x2 +x3 +x4 +x5 = 20, onde xi ´ o n´ mero de bombons e u 20 20 do tipo i. A resposta ´ CR5 = C24 = 10.626. e Solu¸˜es dos Exerc´ co ıcios da Se¸˜o 5.1 ca 1) H´ 6×6 = 36 resultados poss´ a ıveis igualmente prov´veis, em 6 dos quais a 6 1 a soma vale 7. A resposta ´ e = . 36 6 12 2) Basta escolher os times do primeiro grupo, o que pode ser feito de C24 modos. Os dois times em quest˜o ficam juntos quando ficam ambos no a primeiro grupo ou ambos no segundo grupo. Em ambos os casos, isto 10 10 2C22 2.22!.12! 11 pode ser feito de C22 modos. Logo, a resposta ´ 12 = e = . C24 24!10! 23 Outra solu¸˜o: ca Supondo j´ escolhido o grupo do primeiro time, seus 11 companheiros a 11 de grupo podem ser escolhidos de C23 modos. Dentre os grupos assim 10 formados os que tamb´m incluem o segundo time s˜o C22 , j´ que s˜o e a a a formados escolhendo 10 times entre os 22 restantes. Logo, a resposta ´ e 10 C22 11 11 = . C23 23 25
  • 26. 3) Usaremos o fato, j´ provado, de que P (A ∪ B) = P (A) + P (B) − P (A ∩ a B). Temos: P (A∪B ∪C) = P ((A∪B) ∪C) = P (A∪B) + P (C) −P ((A∪B) ∩C) = P (A) + P (B) − P (A ∩ B) + P (C) − P ((A ∪ B) ∩ C). Agora, (A ∪ B) ∩ C = (A ∩ C) ∪ (B ∩ C) e, da´ ı, P ((A ∪ B) ∩ C) = P (A ∩ C) + P (B ∩ C) − P ((A ∩ C) ∩ (B ∩ C)) = = P (A ∩ C) + P (B ∩ C) − P (A ∩ B ∩ C). Substituindo na express˜o anterior, obtemos, finalmente: a P (A ∪ B ∪ C) = P (A) + P (B) + P (C)− −P (A ∩ B) − P (A ∩ C) − P (B ∪ C) + P (A ∩ B ∩ C). 2 4) a) Como A ⊂ A ∪ B, temos P (A ∪ B) ≥ P (A) = . 3 b) Como A ∩ B e B s˜o disjuntos e A ∩ B ∪ B = A ∪ B, temos a P (A ∩ B) + P (B) = P (A ∪ B) e, portanto, 4 P (A ∩ B) = P (A ∪ B) − P (B) = P (A ∪ B) − . 9 2 Mas, do item anterior, ≤ P (A ∪ B) ≤ 1. Da´ ı, 3 2 4 4 2 5 − ≤ P (A ∩ B) ≤ 1 − , ou seja, ≤ P (A ∩ B) ≤ . 3 9 9 9 9 c) Observe que P (A ∩ B) + P (A ∩ B) = P (A), j´ que o conjunto da a direita ´ a uni˜o disjunta dos da esquerda. Da´ P (A ∩ B) = P (A) − e a ı, 2 2 5 2 5 P (A ∩ B) = − P (A ∩ B). Como ≤ P (A ∩ B) ≤ , temos − ≤ 3 9 9 3 9 2 2 1 4 P (A ∩ B) ≤ − , ou seja, ≤ P (A ∩ B) ≤ . 3 9 9 9 5) a) O n´ mero de casos poss´ u ıveis ´ 65 , pois h´ 6 resultados para cada um e a 2 dos 5 dados. O n´ mero de casos favor´veis ´ 6.C5 .5.4.3 = 3600, pois u a e h´ 6 modos de escolher o tipo do par (pode ser de 1, de 2, ..., de 6) a a 2 e h´ C5 modos de escolher os dois dados que formar˜o o par; quanto a aos outros dados, o resultado do primeiro deles pode ser escolhido de 5 modos distintos (deve ser diferente do resultado dos dois primeiros dados), o do segundo pode ser escolhido de 4 modos distintos (deve ser diferente dos anteriores) e, o do terceiro, de 4 modos diferentes. 26
  • 27. 3.600 25 Logo, P (A2) = = ≡ 0, 463. 65 54 b) O n´ mero de casos poss´ u ıveis ´ 65 , pois h´ 6 resultados para cada um e a u a e 2 2 2 dos 5 dados. O n´ mero de casos favor´veis ´ C6 .C5 .C3 .4 = 1800, pois 2 h´ C6 modos de escolher os tipos dos pares (podem ser de 1 e 2, de 1 e a a 2 3, ..., de 5 e 6), h´ C5 modos de escolher os dois dados que formar˜o o a 2 par menor e C3 modos de escolher os dados que formar˜o o par maior. a Para o dado restante, que deve ter resultado diferente do dos demais dados, h´ 4 resultados poss´ a ıveis. 180 25 Logo, P (A3) = 5 = ≡ 0, 231. 6 108 Observa¸˜o: ca Um erro bastante comum ´ contar os casos favor´veis em dobro, racioci- e a nando do modo seguinte: H´ 6 modos de escolher o tipo do primeiro a 2 par, 5 modos de esco lher o tipo do segundo par, C5 modos de escol- 2 her os dois dados que formar˜o o primeiro par, C3 modos de escol- a her os dados que formar˜o o segundo par e h´ 4 modos de escolher a a o resultado do dado restante. Logo, o n´ mero de casos favor´veis ´ u a e 2 2 6.5.C5 .C3 .4 = 3.600. ´ E claro que o erro prov´m da distin¸˜o artificial entre o primeiro e o se- e ca gundo par, que faz com que um par de 2 e um par de 5 seja considerado diferente de um par de 5 e um par de 2. u ıveis ´ 65 , pois h´ 6 resultados para cada um c) O n´ mero de casos poss´ e a 3 dos 5 dados. O n´ mero de casos favor´veis ´ 6.C5 .5.4 = 1200, pois h´ u a e a 6 modos de escolher o tipo da trinca (pode ser de 1, de 2, ..., de 6) e h´ a 3 C5 modos de escolher os trˆs dados que ter˜o resultados iguais; quanto e a aos outros dados, o resultado do primeiro deles pode ser escolhido de 5 modos distintos (deve ser diferente do resultado dos trˆs primeiros e dados) e, o do segundo, de 4 modos distintos (deve ser diferente dos anteriores). 1.200 25 Logo, P (A4) = 5 = ≡ 0, 154. 6 162 d) O n´ mero de casos poss´ u ıveis ´ 65 , pois h´ 6 resultados para cada um e a 4 dos 5 dados. O n´ mero de casos favor´veis ´ 6.C5 .5 = 150, pois h´ 6 u a e a modos de escolher o tipo da quadra (pode ser de 1, de 2, ..., de 6) e a 4 h´ C5 modos de escolher os quatro dados que ter˜o resultados iguais; a 27
  • 28. quanto ao dado restante, seu resultado pode ser escolhido de 5 modos distintos (deve ser diferente do resultado dos quatro primeiros dados). 150 25 Logo, P (A6) = 5 = ≡ 0, 019. 6 1.296 e) O n´ mero de casos poss´ u ıveis ´ 65 e o n´ mero de casos favor´veis ´ 6. e u a e 6 1 Logo, P (A7) = 5 = ≡ 0, 0008. 6 1.296 f) O n´ mero de casos poss´ u ıveis ´ 65 , pois h´ 6 resultados para cada um e a dos 5 dados. H´ dois tipos poss´ a ıveis de seq¨ˆncias: a m´ ue ınima (12345) e a m´xima (23456). A m´ a ınima pode ser formada de 5! = 120 modos distintos, pois h´ 5 modos de escolher o dado cujo resultado ´ 1, 4 a e modos de escolher o dado cujo resultado ´ 2, etc. H´, analogamente, e a 120 modos de formar a seq¨ˆncia m´xima. ue a 240 5 Portanto, P (A8) = 5 = ≡ 0, 031. 6 162 g) O n´ mero de casos poss´ u ıveis ´ 65 , pois h´ 6 resultados para cada e a 3 um dos 5 dados. O n´ mero de casos favor´veis ´ 6.C5 .5 = 300, pois h´ u a e a 6 modos de escolher o tipo da trinca (pode ser de 1, de 2, ..., de 6) e a 3 h´ C5 modos de escolher os trˆs dados que formar˜o a trinca; quanto e a aos outros dados, h´ 5 modos distintos de escolher o resultado comum a deles. 300 25 Logo, P (A5) = 5 = ≡ 0, 039. 6 648 6) Numeremos os v´rtices do pol´ e ıgono de 0 a 2n. Imagine 0 como o v´rtice e mais alto, os v´rtices de 1 a n do lado direito e os v´rtices de n + 1 a e e 2n do lado esquerdo. Podemos pensar que todos os triˆngulos tˆm 0 a e 2n(2n − 1) e a 2 como um dos v´rtices. H´ C2n = = n(2n − 1) modos de 2 selecionar os outros dois v´rtices do triˆngulo. e a Para contar o n´ mero de triˆngulos que contˆm o centro da circun- u a e ferˆncia circunscrita em seu interior, observe inicialmente que a reta e que cont´m o v´rtice i(1 ≤ i ≤ n) e o centro da circunferˆncia corta e e e novamente o pol´ ıgono no ponto m´dio do segmento determinado pelos e v´rtices i + n e i + n + 1 (v´rtice 2n + 1 = v´rtice 0). e e e Um triˆngulo que contenha em seu interior o centro da circunferˆncia a e ser´ necessariamente formado por um v´rtice do lado direito e um a e 28
  • 29. v´rtice do lado esquerdo. Se o v´rtice do lado direito for o v´rtice e e e 1, o do lado esquerdo s´ poder´ ser o v´rtice n + 1 (1 possibilidade); o a e se for o v´rtice 2, poder´ ser qualquer dos v´rtices de n + 1 a n + 2 (2 e a e possibilidades); ...; se for o v´rtice n, poder´ ser qualquer dos v´rtices e a e de n + 1 a 2n (n possibilidades). O n´ mero de casos favor´veis ´ u a e n(n + 1) 1+2+ ···+n = . 2 n+1 A resposta ´e . 2(2n − 1) 7) Imagine o resultado do sorteio como uma fila de 12 lugares: o primeiro lugar corresponde a primeira pessoa sorteada para o primeiro grupo; o ` segundo, a segunda pessoa sorteada para o segundo grupo; ...; o ultimo, ` ´ a ` quarta pessoa sorteada para o terceiro grupo. Colocada a primeira pessoa, h´ 11 posi¸˜es para a segunda, em 3 das quais ela fica no mesmo a co grupo da primeira. 3 A resposta ´ . e 11 8) H´ 12 possibilidades para o signo de cada pessoa, para um total de 124 a possibilidades. Para que n˜o haja coincidˆncias de signos, o signo da a e primeira pessoa pode ser escolhido de 12 modos, o da segunda de 11, o da terceira de 10 e o da quarta de 9, para um total de 12.11.10.9 modos. Assim, a probabilidade de que n˜o haja coincidˆncia de signos a e 12.11.10.9 55 ´ e = e a probabilidade de que n˜o haja coincidˆncias ´ a e e 124 96 55 41 1− = . 96 96 a 4 9) H´ C10 modos de retirar 4 p´s de sapatos. Para retirar 4 p´s, havendo e e nesses 4 p´s exatamente 1 par de sapatos, devemos inicialmente sele- e cionar 1 par (o que pode ser feito de 5 modos) e depois selecionar 2 p´s e vindo de pares diferentes dentre os 4 pares que ainda est˜o no arm´rio. a a 4 Para isso devemos escolher os pares de onde vir˜o esses sapatos (C2 a modos) e, em cada par escolhido, decidir se retiraremos o p´ direito ou e 2 o p´ esquerdo (2 = 4 modos). e 2 5.C4 .4 4 A resposta ´ e 4 = . C10 7 29
  • 30. 10) Para distribuir os sorvetes, devemos escolher as pessoas que receber˜oa 5 sorvetes de creme (C10 modos) e dar sorvetes de chocolate as demais ` (1 modo). Para distribuir os sorvetes, respeitando as preferˆncias, e come¸amos dando sorvetes de creme aos que gostam de creme e de c chocolate aos que gostam de chocolate (1 modo). Em seguida, deve- mos distribuir 2 sorvetes de creme e 3 sorvetes de chocolate a 5 pessoas que n˜o tˆm preferˆncias; para isso, devemos escolher as 2 pessoas que a e e 2 receber˜o sorvetes de creme (C5 modos) e dar sorvetes de chocolate as a ` restantes (1 modo). 2 C5 5 A resposta ´ e 5 = . C10 126 11) As pe¸as do domin´ s˜o formadas por dois, n˜o necessariamente dis- c o a a 2 2 tintos, dos n´ meros 0, 1, 2, 3, 4, 5 e 6. H´ CR7 = C8 = 28 pe¸as e u a c 2 h´ C28 modos de selecionar duas pe¸as de um domin´. Para selecionar a c o duas pe¸as com um n´ mero comum, deve-se primeiramente selecionar c u o n´ mero comum (7 possibilidades) e, depois, selecionar 2 das 7 pe¸as u c 2 que contˆm esse n´ mero comum (C7 possibilidades). e u 2 7C7 7 A resposta ´ e 2 = . C28 18 12) O n´ mero de sorteios poss´ u e 5 ıveis ´ C80 . a) O apostador acerta 3 dezenas quando s˜o sorteadas 3 das 8 dezenas a em que apostou e 2 das 72 em que n˜o apostou. Tais sorteios podem a 3 2 ser efetuados de C8 .C72 modos. 3 2 C8 .C72 1 A resposta ´ e 5 (que ´ aproximadamente igual a e ). C80 168 b) O apostador acerta 4 dezenas quando s˜o sorteadas 4 das 8 dezenas a em que apostou e 1 das 72 em que n˜o apostou. Tais sorteios podem a 4 1 ser efetuados de C8 .C72 modos. 4 1 C8 .C72 1 A resposta ´ e 5 (que ´ aproximadamente igual a e ). C80 4770 c) O apostador acerta 5 dezenas quando s˜o sorteadas 5 das 8 dezenas a 5 em que apostou. Tais sorteios podem ser efetuados de C8 modos. 5 C8 1 A resposta ´ e 5 = . C80 429.286 30
  • 31. 13) Colocada a primeira pessoa na roda, h´ n − 1 posi¸˜es poss´ a co ıveis para a segunda pessoa, das quais 2 s˜o favor´veis a que ela fique junto da a a primeira pessoa. 2 A resposta ´ e . n−1 14) a) H´ n posi¸˜es igualmente prov´veis que a chave “certa” poderia a co a ocupar: ser a primeira a ser testada, a segunda,..., a ultima. A proba- ´ bilidade de ela ocupar a k-´sima posi¸˜o ´ 1/n. e ca e Outra solu¸˜o: ca H´ n! maneiras de ordenar as chaves a serem tentadas. Para formar as a ordena¸˜es que tem a chave certa na k-´sima posi¸˜o, devemos colocar co e ca as n − 1 chaves restantes nas n − 1 posi¸˜es restantes, o que pode ser co feito de (n − 1)! modos. Logo, a probabilidade de que a chave certa (n − 1)! 1 esteja na posi¸˜o k ´ ca e = . n! n b) As primeiras k tentativas podem ser feitas de nk modos (cada chave pode ser escolhida de n modos, j´ que chaves correspondentes a tenta- a tivas frustradas n˜o s˜o descartadas). Para que se acerte na k- ´sima a a e tentativa, as primeiras k − 1 chaves devem ser incorretas (portanto, podem ser escolhidas de (n − 1)k−1 modos) e a de ordem k deve ser a correta (1 modo). Logo, a probabilidade de se acertar na k-´sima e k−1 (n − 1) tentativa ´e . nk a 4 15) a) H´ C12 = 495 modos de selecionar as 4 vagas que n˜o ser˜o ocupadas a a e 9 modos de escolher 4 vagas consecutivas (1 2 3 4, 2 3 4 5, ..., 9 10 11 12). 9 1 A resposta ´e = . 495 55 4 b) H´ C12 = 495 modos de selecionar as 4 vagas que n˜o ser˜o ocupadas. a a a Para contar o n´mero de possibilidades em que n˜o h´ vagas vazias u a a adjacentes, devemos escolher 4 dos 9 espa¸os existentes antes, entre e c 4 depois dos carros para ficarem vazios. Isto pode ser feito de C9 modos. 126 14 Logo a probabilidade de que n˜o haja vagas consecutivas ´ ´ a ee = . 495 55 16) P (A ∩ B ∩ C) = 0, pois A ∩ B ∩ C ⊂ A ∩ C e P (A ∩ C) = 0. 31
  • 32. a) P (A ∪ B ∪ C) = P (A) + P (B) + P (C) − P (A ∩ B) − − P (A ∩ C) − P (B ∩ C) + P (A ∩ B ∩ C) = = 0, 4 + 0, 5 + 0, 3 − 0, 3 − 0 − 0, 1 + 0 = 0, 8. b) P [A − (B ∪ C)] = P (A) − P [A ∩ (B ∪ C)] = = P (A) − P [(A ∩ B) ∪ (A ∩ C)] = = P (A) − P (A ∩ B) − P (A ∩ C) + P [(A ∩ B) ∩ (A ∩ C)] = = P (A) − P (A ∩ B) − P (A ∩ C) + P (A ∩ B ∩ C) = = 0, 4 − 0, 3 − 0 + 0 = 0, 1. c) P [A ∩ (B ∪ C)] = P [(A ∩ B) ∪ (A ∩ C)] = = P (A ∩ B) + P (A ∩ C) − P [(A ∩ B) ∩ (A ∩ C)] = = P (A ∩ B) + P (A ∩ C) − P (A ∩ B ∩ C) = 0, 3 + 0 − 0 = 0, 3. d) P [(A ∩ B) ∪ C] = P (A ∩ B) + P (C) − P (A ∩ B ∩ C) = = 0, 3 + 0, 3 − 0 = 0, 6 18) a) A resposta, naturalmente, ´ 1/2, j´ que, de todos os pares de n´ meros e a u distintos de 1 a 100, em exatamente a metade o primeiro n´mero ´ u e maior do que o segundo. b) O n´ mero total de poss´ u ıveis extra¸˜es ´ 100 × 100 = 10.000, j´ que co e a o bilhete de cada uma das mo¸as pode ser escolhido de 100 modos. Em c 100 destas poss´ıveis extra¸˜es os dois n´ meros s˜o iguais e em metade co u a das restantes, ou seja, em 9900/2 = 4950 delas, o primeiro n´mero ´ u e maior do que o segundo. Logo, a probabilidade de o n´mero de Laura u 4950 ser maior do que o de Telma ´ e = 0, 495. 10000 19) a) S˜o feitos 5 testes quando uma das quatro primeiras pilhas testadas a est´ descarregada, o mesmo ocorrendo com a quinta a ser testada. A a primeira pilha a ser testada pode ser escolhida de 10 modos, a segunda de 9, e assim por diante, para um total de 10.9.8.7.6 modos poss´ ıveis para escolher as 5 primeiras pilhas a serem testadas. Para formar uma sequˆncia de teste em que a segunda defeituosa ´ detectada na 5a ten- e e tativa, devemos escolher a pilha defeituosa que aparece na 5a posi¸˜o ca (2 modos), a posi¸˜o da outra defeituosa (4 modos) e, finalmente, as ca 32
  • 33. pilhas n˜o defeituosas para as demais posi¸˜es (8.7.6 modos). Logo, a a co 2.4.8.7.6 4 probabilidade pedida ´ e = . 10.9.8.7.6 45 b) S˜o efetuados at´ 5 testes quando as pilhas defeituosas aparecem a e nas 5 primeiras tentativas. Como visto no item anterior, h´ 10.9.8.7.6 a modos de se fazer estas tentativas. Para formar aquelas em que as duas defeituosas est˜o entre as testadas devemos escolher a posi¸˜o a ca da primeira pilha defeituosa (5 modos), a da segunda (4 modos) e, finalmente, as pilhas n˜o defeituosas para as outras tentativas (8.7.6 a 5.4.8.7.6 modos). A probabilidade de que sejam feitos at´ 5 testes ´ e e = 10.9.8.7.6 2 2 7 e, portanto, a probabilidade pedida ´ igual a 1 − = . e 9 9 9 c) Para que sejam feitos menos de 4 testes, as duas pilhas defeituosas devem aparecer nos primeiros 4 testes. O n´ mero total de escolhas u para os 4 primeiros testes ´ 10.9.8.7. Para formar uma sequˆncia de e e teste em que as duas defeituosas aparecem nestas 4 tentativas, devemos escolher a posi¸˜o da primeira pilha defeituosa (4 modos), a da segunda ca (3 modos) e, finalmente, as pilhas n˜o defeituosas para as duas outras a 4.3.8.7 2 posi¸˜es (8.7 modos). A probabilidade pedida ´ co e = . 10.9.8.7 15 Solu¸˜es dos Exerc´ co ıcios da Se¸˜o 5.2 ca 1) Sejam X e Y os resultados do primeiro e segundo lan¸amentos, respec- c tivamente. P (X = 3 | X + Y = 7) = P (X = 3, X + Y = 7) 1/6 · 1/6 1 = = = . P (X + Y = 7) 6/36 6 Outra solu¸˜o: ca e a ıveis igualmente prov´veis: X = 1, Y = Se a soma ´ 2, h´ 6 casos poss´ a 6; X = 2, Y = 5; X = 3, Y = 4; X = 4, Y = 3; X = 5, Y = 2; X = 6, Y = 1. Dos seis casos, somente X = 3, Y = 3 ´ favor´vel. A e a 1 resposta ´ . e 6 P (n˜o sabe e acerta) a 2) P (n˜o sabe | acerta) = a = P (acerta) 33